SlideShare a Scribd company logo
1 of 37
Download to read offline
29         th
                  Annual

     In-Training Examination
for Diagnostic Radiology Residents
            Rationales

             February 2, 2006



                    Sponsored by:
              Commission on Education
Committee on Residency Training in Diagnostic Radiology
Section I – Pediatric Radiology




                                               Figure 1A

5.        You are shown two single shot fast spin-echo images (Figures 1A and 1B) from an MR on a
          22-week fetus. What is the MOST LIKELY diagnosis?

     A.   Posterior urethral valves
     B.   Multicystic dysplastic kidneys
     C.   Autosomal recessive polycystic kidney disease
     D.   Bilateral ureteropelvic junction (UPJ) obstruction



                                                                                                    1
                                Diagnostic In-Training Exam 2006
Section I – Pediatric Radiology




                  Figure 1B




2        American College of Radiology
Section I – Pediatric Radiology
Question #5
Rationales:
A. Incorrect. While patients with severe obstruction from posterior urethral valves may also show
   absence of amniotic fluid, posterior urethral valves lead to hydronephrosis, dilated ureters, and a
   markedly distended bladder. Therefore, this choice is incorrect.
B. Incorrect. Although bilateral multicystic dysplastic kidneys would lead to absence of urine in the
   bladder, and to lack of amniotic fluid, this abnormality is rarely bilateral. Further, the kidneys show
   multiple, large macroscopic cysts of different size, which do not communicate. In this case, the kid-
   neys appear solid, since the “cysts” consist of collecting tubules, are microscopic, and the kidneys
   therefore appear solid. Therefore, this choice is incorrect.
C. Correct. The images show symmetrical, markedly enlarged, high signal kidneys, filling the
   abdomen, displacing the diaphragms upwardly, small lungs, and no amniotic fluid. There is no urine
   in the bladder. These findings of enlarged, solid, fluid-intensity kidneys without urine production
   are typical of Autosomal Recessive Polycystic Kidney Disease.
D. Incorrect. Although UPJ obstruction can be bilateral, and can be severe; it is rare that it would be
   severe enough to lead to complete lack of urine production. Furthermore, in those cases,
   hydronephrosis, with dilatation of the calyces, would be evident. In this case, there is no
   hydronephrosis. Therefore, Option D is incorrect.




                                                                                                          3
                             Diagnostic In-Training Exam 2006
Section I – Pediatric Radiology




                                               Figure 2
6.        You are shown an axial T1-weighted image of a newborn term infant (Figure 2). What is the
          MOST LIKELY diagnosis?

     A.   Heterotopia
     B.   Lissencephaly
     C.   Schizencephaly
     D.   Hemimegalencephaly



4                                American College of Radiology
Section I – Pediatric Radiology
Question #6
Rationales:
A. Incorrect. Heterotopia Consists Of Gray Matter In Abnormal Locations, Thought To Be An Arrest
   Of Neuronal Migration Before It Reached The Brain Surface. The Most Common Locations Are
   Subependymal. Single Or Multiple Masses Of Gray Matter May Be Seen In The Subependymal
   Layer Of The Lateral Ventricles Or Subcortical White Matter. This Is Not Present In This Case, And
   This Choice Is Incorrect.
B. Correct. The Image Shows An Abnormally Thickened, Agyric Cortex. Lissencephaly, Or “Smooth
   Brain” Is A Condition In Which There Is A Lack Of Gyri. MRI Features Include Abnormally
   Smooth Cerebral Surface, Cerebral Contour And Gray White Matter Distribution. The Cerebral
   Contour Is Hourglass Shaped With Shallow Sylvian Grooves. There Is Increased Gray Matter.
C. Incorrect. Schizencephaly Is Believed To Result From Disordered Migration, Characterized By
   Abnormal Transcerebral Columns Of Gray Matter. A Cleft Extends Across The Cerebral
   Hemisphere From The Ventricular Surface To The Pial Surface, And Is Lined By The Abnormal
   Gray Matter.
D. Incorrect. Hemimegalencephaly Is A Migrational Disorder Resulting In Hemihypertrophy Of The
   Brain. Heterotopias, Pachygyric Or Agyric Cortex May Be Present.




                                                                                                   5
                            Diagnostic In-Training Exam 2006
Section I – Pediatric Radiology




                                                                         Figure 3B
                     Figure 3A
7.        You are shown T1 and post-contrast, fat-saturation coronal images from an MR examination in a
          3-year-old boy with urinary retention (Figures 3A and 3B). What is the MOST LIKELY diagnosis?

     A.   Metastatic Wilms’ tumor
     B.   Rhabdomyosarcoma
     C.   Germ cell tumor
     D.   Hemorrhagic cystitis




6                                American College of Radiology
Section I – Pediatric Radiology
Question #7
Rationales:
A. Incorrect. Wilms' Tumor Arises Peripherally In The Kidney Rather Than In Central Location And
   Does Not Tend To Spread Via The Ureter. Wilms' Tumor Metastases Most Commonly Occur To
   Local Nodes, Liver And Lung. Tumor Invasion Of The Renal Vein With Spread To The IVC And
   Right Atrium Also Occurs Relatively Frequently.
   Hemorrhage And Necrosis Within Wilms' Tumor Is Common, And Both Subcapsular And
   Perinephric Hematoma Can Result. However, Hemorrhage Into The Collecting System And
   Hematuria Is Quite Uncommon. In This Case No Renal Mass Is Evident. There Is A Large Irregular
   Enhancing Bladder Mass Excluding Bladder Thrombus As A Diagnosis.
B. Correct. The Images Show An Enhancing Polypoid Mass In The Bladder Base And Posterior
   Urethra, With Secondary Bilateral Hydroureteronephrosis, And A Right Perirenal Fluid Collection
   Likely Representing Urinoma. Rhabdomyosarcomas In General Are More Common In Boys Than
   Girls With A Mean Age Of Occurrence Of 7 Years. Genitourinary Rhabdomyosarcoma Most
   Commonly Originates In The Prostate Or Bladder Base In Boys And The Vagina Or Bladder Base
   In Girls. Urinary Retention, Dysuria, Hematuria, Fecal Retention And Vaginal Mass Are The Most
   Common Presenting Features. This Tumor Is Also Known As Sarcoma Botryoides Because Of The
   Irregular Grape-Like Appearance Of The Margins Of The Tumor Extending Into The Bladder. .
C. Incorrect. With The Exception Of Ovarian And Testicular Tumors, Germ Cell Tumors Tend To
   Occur In Very Young Children (Less Than 2 Years). Sacrococcygeal And Head And Neck Tumors
   Are Usually Present In The Newborn. An Occasional Site For Germ Cell Tumor Is In The Vagina In
   Girls, With A Mass That May Mimic Sarcoma Botryoides; However Rhabdomyosarcoma Is The
   Most Common Vaginal Tumor In Young Girls. Sacrococcygeal Teratomas Have Variable Internal
   And External Pelvic Mass Components And Typically Cause Anterior Displacement Of The Rectum
   And Bladder. The Images Depict A Botryoid Tumor Arising From The Bladder Base, And Therefore
   This Choice Is Incorrect.
D. Incorrect. Hemorrhagic Cystitis Can Occur In Association With Viral Or Bacterial Infection As Well
   As Related To Various Drugs (Ether And Hydrogen Peroxide) And Chemotherapeutic Agents
   (Cyclophosphamide). There Is Usually Diffuse, Relatively Smooth Bladder Wall Thickening,
   Although There May Be A More Focal Mass Mimicking A Tumor, And Extensive Clot May Be
   Present In The Bladder. However, One Would Not Expect The Large Heterogeneous, Polypoid,
   Enhancing And Obstructing Mass Seen Here, Therefore This Choice Is Incorrect.




                                                                                                   7
                            Diagnostic In-Training Exam 2006
Section I – Pediatric Radiology




                                                                           Figure 4B
                    Figure 4A
8.        You are shown the frontal and lateral chest radiographs of a previously healthy 6- year-old child
          with cough and fever (Figures 4A and 4B). Which of the options below is the MOST appropriate
          imaging recommendation to the referring physician?

     A.   Follow up chest radiograph
     B.   CT scan
     C.   Ultrasound
     D.   No further imaging is needed




8                                 American College of Radiology
Section I – Pediatric Radiology
Question #8
Rationales: ALL CHOICES ARE CORRECT
The finding consists of a round mass in the left paraspinal, retrocardiac region.
A. The radiographs demonstrate a round opacity in the retrocardiac area, most consistent with a round
   pneumonia in a young child, particularly with the given history. Round pneumonias are common in
   children and are thought to occur because of relatively poor development of collateral air pathways
   for intra-alveolar spread of consolidation. When two views of the chest are obtained, frequently the
   lesion only appears round or mass-like in one plane and not the other. The rounded appearance of
   the mass is also often transient. Round pneumonias are usually associated with pneumococcal infec-
   tion and are most commonly located posteriorly in the perihilar or lower lobe region. In the appro-
   priate clinical situation the mass-like appearance of a round pneumonia should not prompt a pan-
   icked work up for a neoplasm. However, since other lesions can be mistaken for round pneumonia,
   it is considered appropriate to obtain a follow up chest x-ray to document resolution.
B. The images shown demonstrate posterior left retrocardiac opacity. This may be in the middle to pos-
   terior mediastinum or left lower lobe. Differential considerations include round pneumonia, pul-
   monary sequestration, foregut duplication or neurogenic tumor. The clinical history along with the
   location tends to favor a round pneumonia, although no air bronchograms are seen. However air
   bronchograms are only present in about 30% of round pneumonias. No mass effect is seen on ribs,
   mediastinum or airway and the child has mild clinical symptoms. Therefore no urgent additional
   imaging appears necessary. Use of CT scans in children should be carefully considered both in
   order to obtain high quality studies that properly answer clinical questions because of the relatively
   high attendant radiation dose, as well as the frequent need for intravenous contrast and sedation.
   Clinical management and radiographic follow-up with CT obtained electively, if necessary, is a
   more appropriate recommendation.
C. Ultrasound has been utilized in the pediatric chest especially in evaluating mediastinal or juxtamedi-
   astinal lesions. Studies have the advantage of being able to be performed portably in ill patients and
   there is no radiation exposure. However ultrasound is not usually the first choice examination to
   assess an abnormal chest radiographic finding, any interposing aerated lung attenuates the ultra-
   sound beam. The major use of chest ultrasound has been in evaluating pleural fluid collections and
   assessing the cystic or solid nature of accessible lesions.
D. Fever is a common reason for obtaining a chest x-ray, and other lesions may be mistakenly diag-
   nosed as round pneumonia in a child with an acute febrile illness. Therefore it is considered appro-
   priate to obtain a follow up chest x-ray after resolution of the clinical illness. In this case a follow
   up chest x-ray demonstrated that the lesion was unchanged one month later. Further workup
   revealed an esophageal duplication cyst.




                                                                                                          9
                              Diagnostic In-Training Exam 2006
Section I – Pediatric Radiology




                                             Figure 5A




                                             Figure 5B


9.        You are shown two images from a contrast CT image of an 8-year-old boy with leukemia who
          complains of pain after upper endoscopy (Figures 5A and 5B). What is the MOST LIKELY
          diagnosis?

     A.   Chloroma
     B.   Traumatic perforation of the duodenum
     C.   Intramural duodenal hematoma
     D.   Pancreatitis


10                              American College of Radiology
Section I – Pediatric Radiology
Question #9
Rationales:
A. Incorrect. Chloroma, Also Known As Granulocytic Sarcoma, Is A Localized Tumor Of Leukemic
   Cells. In General, Chloromas Are Uncommon, But Are A Diagnostic Consideration With Any Solid
   Mass In A Patient With Leukemia. Chloromas Most Commonly Involve Bone, But May Involve
   Other Organ Systems. A Focal Chloroma Involving Duodenum; However, Would Be Distinctly
   Unusual, Particularly In This Clinical Setting Of Pain After Upper Endoscopy In A Potentially
   Thrombocytopenic Patient. This Option Is Therefore Incorrect.
B. Incorrect. The Mass Is Confined To The Wall Of The Duodenum. No Extraluminal Fluid And No
   Extraluminal Gas Are Seen To Suggest A Perforation Of The Duodenum. This Option Is Therefore
   Incorrect.
C. Correct. Intramural Duodenal Hematoma May Arise From Trauma As Might Occur As A
   Complication Of Upper Endoscopy. Duodenal Hematomas May Also Occur From Blunt Trauma
   (Motor Vehicle Accidents, Bicycle Handle Bar Injuries, Child Abuse) And In Children With
   Bleeding Disorders. Patients With Leukemia May Be Thrombocytopenic Due To The Primary
   Disease Or As Consequence Of Therapy, Further Predisposing To The Development Of Duodenal
   Hematoma. As In This Case, The Mass Is Confined To The Duodenal Wall. The Duodenal Lumen Is
   Compressed By The Hematoma. This Option Is Therefore Correct.
D. Incorrect. Pancreatitis May Be A Complication Of Upper Endoscopy. However, More Commonly,
   Pancreatitis May Be A Complication Of Endoscopic Retrograde Pancreatography (ERCP). In This
   Image, A Mass Is Seen Which Appears To Involve The Duodenum Rather Than The Pancreas. Only
   A Small Portion Of Pancreas Is Displayed On The Higher Of The Two Images, And It Appears
   Normal. There Is No Evidence Of Peripancreatic Edema, Or Significant Inflammatory Changes.
   The Diagnosis Of Pancreatitis Therefore Cannot Be Made From These Images. This Option Is
   Incorrect.




                                                                                             11
                           Diagnostic In-Training Exam 2006
Section I – Pediatric Radiology




           Figure 6A                                            Figure 6B
10.        You are shown plain films and MR (T1 coronal and T1, fat-saturation, post-gadolinium sagittal)
           images of the proximal right tibia in a 12-year-old boy with a swollen and painful right leg
           (Figures 6A through 6C). What is the MOST LIKELY diagnosis?

      A.   Healing stress fracture
      B.   Brodie’s abscess
      C.   Osteoid osteoma
      D.   Osteosarcoma


12                                   American College of Radiology
Section I – Pediatric Radiology




              Figure 6C




                                       13
    Diagnostic In-Training Exam 2006
Section I – Pediatric Radiology
Question #10
Rationales:
A. Incorrect. A Healing Stress Fracture Demonstrates Sclerosis And Periosteal Reaction On Plain Film
   And Some Marrow Edema On MRI. There Is Not Extensive Mass With Enhancement, Extending
   Into The Surrounding Soft Tissues, As Is Seen In This Case. This Option Is Therefore Incorrect.
B. Incorrect. A Brodie’s Abscess Usually Appears As A Well Defined Lytic Lesion With A Sclerotic
   Rim. Marrow Edema Is Seen On MRI. There Is No Soft Tissue Mass. This Option Is Therefore
   Incorrect.
C. Incorrect. An Osteoid Osteoma Has A Central Lucent Nidus And Surrounding Sclerosis. There Is
   No Extension Into The Epiphysis Or Soft Tissue Mass.
D. Correct. The Images Demonstrate An Aggressive, Bone-Producing Mass Involving The Proximal
   Right Tibia With Abnormal Periosteal Reaction. The MRI Demonstrates A Solid, Enhancing Lesion
   Replacing The Marrow Cavity, Crossing The Medial Physis, And An Enhancing Soft Tissue
   Extension Beyond The Bone. The Most Likely Diagnosis Is Osteosarcoma.




14                            American College of Radiology
Section I – Pediatric Radiology
11.        A newborn infant presents on the second day of life with a distended abdomen and bilious eme-
           sis. A radiograph of the abdomen demonstrates markedly distended intestinal loops throughout
           the abdomen. What is the BEST next diagnostic examination?

      A.   Upper GI study
      B.   Contrast enema
      C.   CT of the abdomen with IV contrast
      D.   Ultrasound examination of the abdomen

Question #11
Rationales:
A. Incorrect. The Case Describes A Distal Bowel Obstruction In A Newborn. An Upper GI Study
   Would Not Be Warranted In Cases Of A Newborn With A Distal Bowel Obstruction. The Correct
   Test Of Choice Would Be A Contrast Enema, Preferably Performed With Water Soluble, Isotonic
   Contrast In Case Of A Perforation.
B. Correct. The Case Describes A Distal Bowel Obstruction In A Newborn. The Correct Test Of
   Choice Would Be A Contrast Enema, Preferably Performed With Water Soluble, Isotonic Contrast
   In Case Of A Perforation.
C. Incorrect. The Case Describes A Distal Bowel Obstruction In A Newborn. Abdominal CT Plays No
   Role In The Workup. The Correct Test Of Choice Would Be A Contrast Enema, Preferably
   Performed With Water Soluble, Isotonic Contrast In Case Of A Perforation.
D. Incorrect. The Case Describes A Distal Bowel Obstruction In A Newborn. Abdominal Sonography
   Plays No Role In The Workup, And The Gas-Filled Bowel Would Render The Examination Difficult
   At Best. The Correct Test Of Choice Would Be A Contrast Enema, Preferably Performed With
   Water Soluble, Isotonic Contrast In Case Of A Perforation.




                                                                                                     15
                                Diagnostic In-Training Exam 2006
Section I – Pediatric Radiology
         Concerning holoprosencephaly, which one is TRUE?
12.

      A. The fornix and septum pellucidum are absent in all forms of holoprosencephaly.
      B. The posterior portion of the corpus callosum is absent in lobar holoprosencephaly.
      C. 10% of cases with holoprosencephaly are associated with cytogenetic abnormalities.
      D. Cebocephaly is most common in patients with lobar holoprosencephaly.

Question #12
Rationales:
A. Correct. While The Severity Of Holoprosencephaly Varies From Severe Alobar To Lobar, By
   Definition, All Forms Of Holoprosencephaly Do Not Have A Fornix Or Septum Pellucidum, And
   May Have Hypoplasia Of The Pituitary And Optic Nerves.
B. Incorrect. The Cerebral Vesicles Cleave In A Dorsofrontal Direction. In Patients With Lobar And
   Semilobar Holoprosencephaly, The Posterior Portion Of The Corpus Callosum Is Present While The
   Anterior Portion Is Absent; The Entire Corpus Callosum May Be Present In Lobar
   Holoprosencephaly.
C. Incorrect. Up To 40% Of Cases Are Linked To Chromosomal Anomalies, 20% With Syndromes,
   With Only 40% Cases Isolated.
D. Incorrect. Severe Facial Anomalies, Such As Cyclopia, Cebocephaly And Ethmocephaly, Are Most
   Likely In Patients With Alobar Holoprosencephaly. However, A Normal Face Does Not Exclude A
   Severe Form Of Holoprosencephaly.




16                              American College of Radiology
Section I – Pediatric Radiology
           Which one of the following findings is seen in most patients with Dandy
13.
           Walker malformation?

      A.   Intact vermis
      B.   Small cisterna magna
      C.   Elevated tentorium cerebelli
      D.   Agenesis of the corpus callosum

Question #13
Rationales:
A. Incorrect. By Definition, The Most Characteristic Finding Of Dandy Walker Malformation Is
   Complete Or Partial Absence Of The Cerebellar Vermis.
B. Incorrect. Fourth Ventricle Communicates With An Enlarged Cisterna Magna Bordered By The
   Cerebellar Hemispheres.
C. Correct. The Tentorium And Torcula Are Elevated With Dandy Walker Malformation, Best
   Evaluated On Midline Sagittal Images Of The Brain. On Plain Films, The Anomaly Is Characterized
   By Lambdoid-Torcular Inversion.
D. Incorrect. Dysgenesis Of The Corpus Callosum Occurs In Only 20-25% Of Patients. Gyral
   Abnormalities Also Occur In A Small Number Of Patients With Dandy Walker Malformation.




                                                                                               17
                                Diagnostic In-Training Exam 2006
Section I – Pediatric Radiology
           Concerning periventricular leukomalacia, which one is CORRECT?
14.

      A.   Injury is secondary to venous infarction.
      B.   Areas of cystic necrosis develop 1-3 weeks after injury.
      C.   Infants are at greatest risk for PVL at 36-38 weeks gestation.
      D.   Ultrasound is sensitive in the identification of the acute lesions.

Question #14
Rationales:
A. Incorrect. Injury is thought to be arterial watershed infarction of the premature infant’s periventricu-
   lar white matter. Venous infarction is believed to be the mechanism of the parenchymal hemorrhagic
   component in Grade IV germinal matrix hemorrhage.
B. Correct. The multicystic periventricular components, characteristic at sonography, represent areas
   of cystic necrosis in ischemic areas, and develop 1-3 weeks after the acute injury, after resolution of
   the initially echodense lesions.
C. Incorrect. Injury typically occurs in preterm infants less than 32 weeks gestation.
D. Incorrect. Ultrasound is often insensitive in the acute stage, with as many as 50% of initial studies
   in the first few days following injury reported as normal in appearance.




18                                  American College of Radiology
Section I – Pediatric Radiology
15.        Which one of the following is characterized by the finding of a microcolon?

      A.   Duodenal atresia
      B.   Meconium plug syndrome
      C.   Meconium ileus
      D.   Short-segment Hirschsprung’s disease

Question #15
Rationales:
A. Incorrect. In patients with duodenal atresia, sufficient succus entericus is produced to allow normal
   colonic caliber.
B. Incorrect. Mecomiun plug syndrome is associated with dysmotility of the colon; infants are typical-
   ly full term, often with maternal history of preeclampsia. Colon is normal in caliber and filled with
   meconium plugs. Once the meconium is extruded, the patient is relieved of symptoms.
C. Correct. Meconium ileus, associated with cystic fibrosis, is the result of inspissation of tenacious
   meconium in the ileum, which prevents passage of meconium into the colon. The colon is not used
   prenatally, resulting in a microcolon.
D. Incorrect. Hirschsprung’s disease is characterized by a distal aperistaltic segment, typically of nor-
   mal caliber, with a more dilated proximal bowel as the transition zone develops. The abnormal seg-
   ment may demonstrate a serrated appearance, believed to result from uncoordinated, aperistaltic
   contractions. Suction biopsy can confirm the diagnosis.




                                                                                                      19
                                Diagnostic In-Training Exam 2006
Section I – Pediatric Radiology
           Concerning the presence of a perirenal fluid collection in a newborn infant, which one is
16.
           CORRECT?

      A.   This finding is associated with posterior urethral valves.
      B.   The finding represents sympathetic transudation of fluid due to obstruction.
      C.   Secondary infection is not a significant concern.
      D.   Early post-contrast CT or MR images are diagnostic.

Question #16
Rationales:
A. Correct. Long standing high grade obstruction in utero is associated with renal dysplasia and poor
   renal function. The presence of loculated or free urine extravasation from the kidney of a newborn
   infant is most commonly associated with posterior urethral valves, and is secondary to forniceal
   rupture and decompression into the perirenal space, at times extending into the peritoneal cavity,
   presenting as urinary ascites. This finding has been associated with relative preservation of function
   in the kidney, probably due to decompression of the obstruction.
B. Incorrect. This fluid collection results from forniceal rupture of a calyx and extravasation of urine
   forming a subcapsular urinoma. Subcapsular, perinephric and paranephric urinomas can occur in
   association with urinary obstruction, including posterior urethral valves, ureteropelvic junction
   obstruction, neurogenic bladder and bladder tumors or after trauma to the urinary tract. The collec-
   tion may rupture into the peritoneal space producing urinary ascites.
C. Incorrect. Secondary infection of fluid collections in a newborn is always of concern, especially
   when there is underlying obstruction and/or reflux.
D. Incorrect. If a contrasted examination (CT or MRI) were done, delayed images would be most use-
   ful in demonstrating contrast accumulation within the urinoma, confirming the connection to the
   urinary tract. In the setting of posterior urethral valves, however, upper tract contrast examinations
   are unnecessary for diagnosis.




20                                 American College of Radiology
Section I – Pediatric Radiology
17.        Concerning leakage radiation for a unit used for pediatric chest filming, what is the maximum
           leakage radiation allowed at 1 meter from the x-ray tube source when the system is operated at the
           maximum continuous allowable tube current and kilovoltage?

      A.   2 mR/week
      B.   10 mR/hour
      C.   100 mR/hour
      D.   No specified limits

Question #17
Rationales:
A. Incorrect. This is maximum permissible dose limit outside an x-ray room.
B. Incorrect. See correct answer.
C. Correct. According to NCRP report no. 91, the allowed maximum limit for leakage radiation is 100
   mR/hr at 1 meter when the system is operated at maximum continuous mA and kVp.
D. Incorrect. See correct answer.




                                                                                                         21
                                 Diagnostic In-Training Exam 2006
Section I – Pediatric Radiology
18.        Which of the following radiographic findings is MOST useful in excluding the diagnosis of
           hyaline membrane disease in a newborn infant?

      A.   Pleural effusion
      B.   Hyperinflation
      C.   Unossified humeral epiphyses
      D.   Asymmetric pulmonary opacity

Question #18
Rationales:
A. Incorrect. While pleural effusion is an uncommon finding in hyaline membrane disease, its presence
   or absence is not specific in excluding the diagnosis. Small pleural effusion may be seen in transient
   tachypnea of the newborn which can co-exist with hyaline membrane disease, especially on early
   radiographs. Although in the appropriate clinical setting, the presence of diffuse ground glass opaci-
   ty with accompanying pleural effusion should raise the question of infection, particularly group B
   streptococcal pneumonia, pleural effusion alone cannot exclude hyaline membrane disease.
B. Correct. The underlying cause of hyaline membrane disease is deficiency of surfactant; this
   lipoprotein complex produced by Type II pneumocytes coats alveoli and prevents them from col-
   lapsing in expiration. Diffuse alveolar atelectasis with pulmonary underaeration is the hallmark of
   this disease. Increased lung volumes would strongly suggest excluding hyaline membrane disease as
   a diagnostic possibility. With early intubation and administration of exogenous surfactant, character-
   istic radiographic findings may be less marked and more transient.
C. Incorrect. Ossification of humeral epiphyses may be used as a sign of skeletal maturity on a chest
   radiograph. Ossified humeral epiphyses are not seen in infants less than thirty-six weeks of gesta-
   tion and are present in approximately 40% of term infants. Therefore the presence of humeral epi-
   physes would diminish the likelihood of hyaline membrane disease, a disease of prematurity.
   Unossified epiphyses would not be useful in excluding the disease.
D. Incorrect. While diffuse symmetric ground glass opacity is the characteristic feature of hyaline
   membrane disease, some variation of this pattern is not uncommon. Lower lobe involvement tends
   to be more marked than upper lobe; opacity may be more marked on the right than on the left side.
   After surfactant administration there is often asymmetric clearing of the lungs. Therefore, the pres-
   ence of asymmetry does not exclude the diagnosis of hyaline membrane disease.




22                                American College of Radiology
Section I – Pediatric Radiology
           Concerning the thymus in childhood, which one is CORRECT?
19.

      A.   Hodgkin’s disease rarely involves the thymus.
      B.   Normal thymus obscures the great vessels at sonography.
      C.   Thymic cysts may occur in the neck.
      D.   Normal thymus is bright on T1- and dark on T2-weighted MR images.

Question #19
Rationales:
A. Incorrect. Hodgkin’s disease often occurs in the anterior mediastinum and typically infiltrates the
   thymus. Bulky adjacent mediastinal adenopathy usually occurs. On CT imaging, the thymus is
   enlarged, nodular and heterogeneous with areas of necrosis frequently present, as opposed to the
   smooth homogenous normal thymus. Compression of the airway is often a significant concern.
B. Incorrect. The normal thymus is of uniform, midi-level echogenicity, and offers an excellent win-
   dow into the mediastinum, by displacing surrounding air-filled lung.
C. Correct. Thymic cysts are most commonly found in thymic rests in the neck, rather than in the
   main gland. These may present acutely due to infection or hemorrhage. They may be associated
   with bone marrow aplasia and have been reported in patients with HIV and treated lymphoma.
D. Incorrect. On MR images the normal thymus in childhood is relatively dark (slightly hyperintense
   to muscle) on T1 weighted images and has uniform bright signal on T2 weighted sequences. In
   younger infants the thymus has a quadrilateral shape with convex borders, becoming more triangu-
   lar in children over five years of age. Fatty infiltration of the thymus is usually not seen until the
   late teenage years or adulthood.




                                                                                                       23
                               Diagnostic In-Training Exam 2006
Section I – Pediatric Radiology
         Concerning imaging of a child with a bronchial foreign body, which one is CORRECT?
20.

      A. A decubitus radiograph with the affected side up will be helpful in demonstrating ipsilateral air
         trapping.
      B. A nuclear medicine ventilation perfusion scan will likely demonstrate symmetric lung perfusion
         but asymmetric ventilation.
      C. On fluoroscopy the mediastinum will shift away from the affected side in expiration.
      D. The majority of foreign bodies are radiopaque and visible on chest radiographs.

Question #20
Rationales:
A. Incorrect. When placed in the decubitus position, the dependent (down) lung deaerates and the non-
   dependent lung expands. Therefore to demonstrate air trapping i.e. lack of dependent deflation, the
   useful decubitus view is with the affected side down.
B. Incorrect. When the lung is abnormally ventilated there is also decreased perfusion due to reflex
   vasoconstriction. There may be very little perfusion of the affected side when significant air trap-
   ping is present.
C. Correct. This is one of the cardinal observations in fluoroscopic evaluation of a foreign body. On
   inspiration the mediastinum is in its normal location i.e. central. With expiration the mediastinum
   shifts towards the normal lung which deflates, and away from the persistently hyperinflated abnor-
   mal lung. Other fluoroscopic observations include lack of deflation and decreased diaphragmatic
   excursion on the affected side
D. Incorrect. Only a small number of foreign bodies are radiopaque. The most common endobronchial
   foreign bodies are ingested foods, such as peanuts and raw carrots. Therefore the radiographic fea-
   tures rely on secondary findings, predominantly air trapping distal to the foreign body due to partial
   obstruction of the affected airway. With a larger or more long standing foreign body, pulmonary
   opacity due to atelectasis or post obstructive pneumonia may occur.




24                               American College of Radiology
Section I – Pediatric Radiology
         Concerning Tetralogy of Fallot, which one is CORRECT?
21.

      A. A right-sided aortic arch is present in approximately 60% of cases.
      B. The four features consist of ASD, VSD, overriding aorta and right ventricular outflow tract
         obstruction.
      C. Radiographs typically demonstrate marked cardiomegaly with a “boot shaped” heart.
      D. Right-sided obstruction may include infundibular, valvular and peripheral pulmonic stenoses.

Question #21
Rationales:
A. Incorrect. Tetralogy of Fallot is one of the most frequent lesions to be associated with a right-sided
   aortic arch. This occurs however in approximately 20 - 30% of cases. Other cardiac lesions associat-
   ed with a right-sided aortic arch include truncus arteriosus (30%), tricuspid atresia (10 - 15%) and
   double outlet right ventricle (12%).
B. Incorrect. ASD is not one of the four cardinal features in Tetralogy of Fallot. Right ventricular
   hypertrophy is the fourth feature in addition to VSD, overriding aorta and right ventricular outflow
   obstruction. ASD occurs in approximately 15% of cases; when present the entity is termed
   Pentalogy of Fallot.
C. Incorrect. Although the quot;boot shapedquot; heart is the classically described configuration in Tetralogy of
   Fallot, preoperatively the heart is usually normal in size, or only mildly enlarged. The “boot-shaped
   heart” is produced by the combination of a concave main pulmonary artery segment and an elevated
   cardiac apex, indicative of right ventricular hypertrophy without dilatation and cardiac enlargement.
D. Correct. Right-sided obstruction is often at multiple levels. Typically there is infundibular obstruc-
   tion without or with valvular stenosis, or even atresia. Peripheral pulmonary artery coarctation, uni-
   lateral hypoplasia or absence of a pulmonary artery also occurs, most often on the left side.
   Conversely there may be complete absence of the pulmonary valve leaflets with marked pulmonic
   regurgitation and very large central pulmonary arteries.




                                                                                                        25
                               Diagnostic In-Training Exam 2006
Section I – Pediatric Radiology
           Concerning slipped capital femoral epiphysis, which one is CORRECT?
22.

      A.   The frog lateral radiograph is contraindicated
      B.   Produces knee pain in one quarter of patients
      C.   Is bilateral in 75 – 80% of cases
      D.   Is most common in athletes

Question #22
Rationales:
A. Incorrect. The femoral head slips posteriorly and medially. The frog lateral view is essential to
   detect the posterior component of the slip, which may be the major component in some patients.
B. Correct. Patients with slipped capital femoral epiphysis usually present with hip or groin pain;
   however, in approximately 25% of patients pain is referred to the ipsilateral knee. These patients
   may present with knee pain alone or knee pain in combination with hip/groin pain. In a patient in
   the typical age range for slipped capital femoral epiphysis, unexplained knee pain should prompt
   consideration for slipped capital femoral epiphysis.
C. Incorrect. Slipped capital femoral epiphysis is bilateral in approximately one quarter to one third of
   patients, much less than 75-80%.
D. Incorrect. Slipped capital femoral epiphysis is most common in obese adolescents.




26                                 American College of Radiology
Section I – Pediatric Radiology
           Concerning Ewing sarcoma of bone, which one is CORRECT?
23.

      A.   It is most common in children under 5 years of age.
      B.   It is the most common primary bone malignancy in childhood.
      C.   The more aggressive tumors produce more osteoid.
      D.   Histologically it is identical to primitive neuroectodermal tumor (PNET).

Question #23
Rationales:
A. Incorrect. Although Ewing sarcoma can occur in a child under 5 years of age, it is relatively rare in
   this age group. Ewing sarcoma is most common in the second decade of life.
B. Incorrect. Ewing sarcoma is the second most common primary bone malignancy in childhood.
   Osteosarcoma is considerably more common.
C. Incorrect. Ewing sarcoma does not produce osteoid. Osteoid is produced by osteosarcoma. A scle-
   rotic response may be seen within the host bone and up to 15% of Ewing sarcomas will thus appear
   sclerotic at presentation; however, the tumor itself does not produce osteoid.
D. Correct. Ewing sarcoma and PNET are histologically identical. Moreover, they are cytogenetically
   identical and now considered by pathologists to be the same tumor.




                                                                                                     27
                                 Diagnostic In-Training Exam 2006
Section I – Pediatric Radiology
24.        Concerning the conversion of hematopoietic bone marrow to fatty bone marrow in the femur of a
           child, which portion of the femur is the LAST to convert?

      A.   Proximal femoral epiphysis
      B.   Proximal femoral metaphysis
      C.   Diaphysis
      D.   Distal femoral metaphysis

Question #24
Rationales:
A. Incorrect. The epiphyses almost immediately convert from hematopoietic marrow to fatty marrow
   as soon as they are ossified.
B. Correct. The epiphyses convert to fatty marrow soon after they ossify. Diaphysis precedes metaph-
   ysis and distal precedes proximal. Therefore, within the femurs, the proximal metaphyses are the
   last portion of the bone to convert from hematopoietic marrow to fatty marrow. Commonly in
   teenagers, residual patchy hematopoietic marrow is still present within the proximal femoral meta-
   physis on MR imaging.
C. Incorrect. The diaphysis converts from hematopoietic marrow to fatty marrow before the metaphy-
   ses.
D. Incorrect. Conversion proceeds from distal to proximal. The distal femoral metaphyses therefore
convert from hematopoietic marrow to fatty marrow prior the proximal femoral metaphyses.




28                                American College of Radiology
Section I – Pediatric Radiology
25.        To keep patient dose as low as reasonably achievable for digital radiography examinations using
           a CR or DR detector, the mAs is selected to provide acceptable:

      A.   image contrast.
      B.   optical density.
      C.   luminance on the image monitor.
      D.   signal to noise ratio.

Question #25
Rationales:
A. Incorrect. Digital detectors are inherently limited by signal to noise ratio, and contrast is variable
   according to digital grayscale translation tables (windowing and leveling the image).
B. Incorrect. There is no optical density measure with digital images. For analog screen-film images
   this, of course, is what is required.
C. Incorrect. The luminance of the monitor is independent of the characteristics of the image. Also, for
   digital images, the overall signal intensity of the output image is scaled so that the proper range of
   signals is displayed (or at least they can be adjusted to the appropriate luminance range)
D. Correct. Digital detectors are inherently signal to noise ratio (SNR) limited, because of the ability
   to vary the image display characteristics by post-processing (contrast enhancement, spatial resolu-
   tion enhancement, etc.) The mAs is linearly related to the number of x-ray photons on the patient
   and also to the number of x-ray photons absorbed by the x-ray detector. Therefore, the SNR (N/√N
   = √N) is determined by the number of x-rays detected and converted to an output signal. Even
   though digital systems have the ability to enhance contrast, noise will also be enhanced and the SNR
   will remain constant. So, the ability to detect a subtle finding in a noisy background for a digital
   image depends on the required SNR for a given examination. The mAs should be chosen at a level
   that gives just enough (not too much) radiation such that the SNR will be high enough to make the
   diagnosis.




                                                                                                            29
                                Diagnostic In-Training Exam 2006
Section I – Pediatric Radiology
           Which one of the following is a characteristic feature of renal duplication in girls?
26.

      A.   Reflux into the upper system
      B.   Ectopic lower pole ureter
      C.   Incontinence
      D.   Duplication of the uterus

Question #26
Rationales:
A. Incorrect. Although reflux may occur into either the upper or lower moieties of an uncomplicated
   duplication, the typical consequence of duplication is obstruction of the ectopic upper pole ureter,
   and reflux into the lower moiety, likely secondary to interference of the abnormal upper pole ureter
   with the normal antireflux tunnel mechanism of the orthotopic lower pole.
B. Incorrect. The lower pole ureter inserts normally at the trigone. The upper pole ureter arises above
   the normal ureteric bud, and therefore inserts ectopically, medially and inferiorly within the bladder,
   or along the course of the mesonephric duct. Therefore, option B is false.
C. Correct. The ureters are derivatives of the mesonephric duct, and therefore ectopic ureters insert
   along its derivatives or remnants. In boys, the ectopic ureter will always insert above the sphincter,
   medially into the bladder, or into the seminal vesicles. However, in girls with duplex kidney, the
   ectopic ureter may insert below the sphincter, into the urethra, or into the vagina, resulting in uri-
   nary incontinence. This is characterized by constant dampness. Therefore, option C is correct.
D. Incorrect. Duplication of the uterus, with obstruction of one of the two systems, is typically associ-
   ated with ipsilateral renal agenesis or dysgenesis. However, renal duplications are not typically
   associated with duplication of the uterus.




30                                 American College of Radiology
Section I – Pediatric Radiology
           Regarding duodenal obstruction, which one is CORRECT?
27.

      A.   Plain films may be normal in patients with volvulus.
      B.   Nonbilious vomiting excludes the diagnosis.
      C.   Atresia is secondary to intrauterine ischemia.
      D.   Windsock duodenum is typically found in neonates.

Question #27
Rationales:
A. Correct. Volvulus obstructs at the third portion of the duodenum, and presents with bilious vomit-
   ing. Volvulus occurs about the axis of the superior mesenteric artery, and leads to ischemia of its
   territory, from the duodenojejunal junction to the splenic flexure. Therefore, insufficient time elaps-
   es for marked duodenal dilatation to occur, and the plain films may be normal, or demonstrate rela-
   tive paucity of distal gas with longer or more severe obstruction. Absence of bowel distension may
   lead to a false sense of security and diminished concern for obstruction.
B. Incorrect. Although most cases of duodenal atresia or stenosis occur distal to the ampulla of Vater,
   the obstruction may lie proximal to the ampulla in a minority of patients, in whom the emesis may
   be nonbilious. Therefore, nonbilious vomiting does not exclude the diagnosis, and this option is
   incorrect.
C. Incorrect. Although small bowel and colonic atresias are considered to be secondary to an intrauter-
   ine ischemic event, duodenal atresia is believed to be the result of failure of recanalization.
   Therefore this option is incorrect.
D. Incorrect. Windsock duodenum is the result of stretching of duodenal membrane in patients with
   membranous duodenal stenosis. Therefore, it is typically seen in older patients.




                                                                                                       31
                                 Diagnostic In-Training Exam 2006
Section I – Pediatric Radiology
           Regarding pyloric stenosis, which one is TRUE?
28.

      A.   It is present at birth.
      B.   Affected infants present with paradoxical aciduria.
      C.   Paternal history is a greater risk factor than maternal history.
      D.   Affected infants have no appetite.

Question #28
Rationales:
A. Incorrect. Pyloric stenosis develops after birth, and typically presents in infants at 4-12 weeks of
   age.
B. Correct. Vomiting leads to dehydration and waste of sodium, potassium and hydrochloric acid.
   Renal mechanisms supervene, with conservation of sodium at the expense of hydrogen ion, leading
   to a paradoxical aciduria in a patient with hypochloremic alkalosis.
C. Incorrect. Male and female children of affected mothers carry a 20% and 7% respective risk of
   developing the condition, whereas male and female children of affected fathers carry a respective
   risk of 5% and 2.5%.
D. Incorrect. Despite the distended stomach, the infants are malnourished, and exhibit a voracious
   appetite, typically gnawing at their hands in an effort to obtain nourishment.




32                                  American College of Radiology
Section I – Pediatric Radiology
29.        Which of the following conditions is associated with pulmonary hypoplasia?

      A.   Bochdalek hernia
      B.   Potter syndrome
      C.   Vascular ring
      D.   Tracheal bronchus

Question #29
Rationales: A & B ARE BOTH CORRECT
A. Correct. The posterolateral Bochdalek diaphragmatic hernia is usually large and associated with
   considerable mass effect on the developing lung. The anteromedial Morgagni hernia is typically
   small and most often asymptomatic. Bochdalek hernia is strongly associated with pulmonary
   hypoplasia whereas hypoplasia is usually not a feature of the Morgagni defect. Occasionally
   Morgagni hernia may be large or bilateral often with herniation into the pericardial sac with car-
   diorespiratory compromise.
B. Correct. Potter syndrome was originally described as renal agenesis in association with fetal anuria
   and oligohydramnios. Bilateral pulmonary hypoplasia is thought to be caused by extrinsic compres-
   sion of the fetal thorax by the uterus in association with decreased amniotic fluid. Other causes of
   olgohydramnios including bilateral urinary obstruction, bilateral cystic or dysplastic kidneys and
   prolonged amniotic fluid leak have also been found to be associated with pulmonary hypoplasia.
   Thoracic cage compression, most commonly due to bone dysplasia and intrathoracic lung compres-
   sion, notably associated with congenital diaphragmatic hernia, pleural fluid or a large thoracic or
   abdominal mass are also causes of bilateral pulmonary hypoplasia. Additional etiologic factors in
   bilateral pulmonary hypoplasia are conditions associated with decreased fetal breathing (e.g.
   myotonic dystrophy, myasthenia gravis, hypotonia) and decreased pulmonary blood flow, such as in
   Tetralogy of Fallot and pulmonary atresia.
C. Incorrect. Vascular ring is a result of abnormal vascular and ligamentous mediastinal structures,
   with encirclement of the trachea and esophagus. The symptomatic vascular ring typically causes
   compression of the trachea and presents with respiratory distress and repeated episodes of pneumo-
   nia. However, this abnormality is not associated with pulmonary hypoplasia.
D. Incorrect. A tracheal bronchus describes the situation where the right upper lobe, or one of its seg-
   ments, is supplied directly from the trachea, rather than as a branch of the right mainstem bronchus.
   When the bronchus supplies the entire lobe, it is termed a “bronchus sui” or pig bronchus, as this is
   the normal porcine anatomical arrangement. The tracheal bronchus may be asymptomatic and dis-
   covered incidentally, or may be associated with repeated right upper lobe infections. Its incidence is
   reported as 0.1 – 2% of the population, and is associated with rib anomalies and Down syndrome.
   However, it is not associated with pulmonary hypoplasia.




                                                                                                        33
                                Diagnostic In-Training Exam 2006
Section I – Pediatric Radiology
30.        Regarding pulmonary sling, which of the following is TRUE?

      A.   The right pulmonary artery arises aberrantly from the left.
      B.   The abnormal artery crosses posterior to the esophagus.
      C.   Pulmonary expansion on the chest radiograph is usually normal.
      D.   It can be complicated by tracheal stenosis.

Question #30
Rationales:
A. Incorrect. The pulmonary artery sling consists of abnormal origin of the left pulmonary artery from
   the right. The aberrant left artery does not arise from the main pulmonary artery at the bifurcation,
   but from the right branch; it then courses posteriorly, and turns to the left between the trachea and
   the esophagus to the left hilum. Therefore, this choice is incorrect.
B. Incorrect. As stated above, the course of the aberrant pulmonary artery lies between the trachea and
   the esophagus, therefore it courses anterior to the esophagus. This is in contradistinction to vascular
   rings, where the abnormal vessel courses posterior to the esophagus, encircling both the trachea and
   esophagus within the vascular ring.
C. Incorrect. Plain chest radiographs on patients with pulmonary artery sling typically show abnormal
   inflation, with hyperinflation or asymmetrical inflation of the lungs. There may be a low position of
   the left hilum, as the normal epibronchial left pulmonary artery is absent. Anterior bowing of the
   trachea is often seen on the lateral view, as the aberrant vessel courses between the trachea and the
   esophagus.
D. Correct. Tracheal stenosis may occur at the site of the crossing vessel, or may be part of a more
   generalized anomaly with complete cartilage rings. The anomaly is typically symptomatic early in
   life, with infants presenting with respiratory stridor, feeding difficulties, and repeated respiratory
   infections.




34                                American College of Radiology
Section I – Pediatric Radiology
31.        Which of the following is a Salter IV fracture?

      A.   Lateral condylar fracture of the distal humerus
      B.   Avulsion of the medial epicondyle of the distal humerus
      C.   Juvenile Tillaux fracture
      D.   Bucket handle fracture of non-accidental trauma

Question #31
Rationales: A & C ARE BOTH CORRECT
A. Correct. The lateral condylar fracture extends through distal humeral metaphysis, distal humeral
   growth plate and distal humeral epiphysis. This is a Salter IV fracture. When the epiphyseal portion
   of the fracture passes through cartilage and is not seen on radiographs, it can be delineated by MR
   or ultrasound. If the fracture is complete there may be displacement of the lateral condylar frag-
   ment. If the fracture through the epiphyseal cartilage is incomplete, there may be minimal or no dis-
   placement of the lateral condylar fragment.
B. Incorrect. An avulsion fracture of the medial epicondyle goes through the growth plate of the medi-
   al epicondylar apophysis. This is a Salter I fracture.
C. Correct. Juvenile Tillaux fracture is a Salter III fracture. The fracture line course through epiphysis,
   usually in a sagittal or oblique sagittal plane, and through the lateral portion of growth plate. This
   fracture occurs near puberty when the distal tibial growth plate is beginning to fuse. Fusion begins
   medially. The Tillaux fracture involves the lateral aspect of the growth plate which is not yet fused.
D. Incorrect. The “bucket handle” fracture of non-accidental trauma does not involve the growth plate
   and is not classified by the Salter-Harris system. Histologically, Kleinman et al have shown that
   this fracture is a transverse fracture through the metaphysis close to the growth plate.




                                                                                                       35
                                 Diagnostic In-Training Exam 2006
Section I – Pediatric Radiology




36        American College of Radiology

More Related Content

What's hot

23205065
2320506523205065
23205065radgirl
 
23205048
2320504823205048
23205048radgirl
 
23204964
2320496423204964
23204964radgirl
 
23205032
2320503223205032
23205032radgirl
 
23204973
2320497323204973
23204973radgirl
 
23205029
2320502923205029
23205029radgirl
 
23204961
2320496123204961
23204961radgirl
 
23204958
2320495823204958
23204958radgirl
 
23205011
2320501123205011
23205011radgirl
 
23204955
2320495523204955
23204955radgirl
 
23204943
2320494323204943
23204943radgirl
 
23204916
2320491623204916
23204916radgirl
 
23204995
2320499523204995
23204995radgirl
 
23205001
2320500123205001
23205001radgirl
 
23205008
2320500823205008
23205008radgirl
 
Radiology most important signs sample questions based on neet pg , usmle, pla...
Radiology most important signs sample questions based on neet pg , usmle, pla...Radiology most important signs sample questions based on neet pg , usmle, pla...
Radiology most important signs sample questions based on neet pg , usmle, pla...Medico Apps
 
23204946
2320494623204946
23204946radgirl
 
Previous year question on bone cyst based on neet pg, usmle, plab and fmge or...
Previous year question on bone cyst based on neet pg, usmle, plab and fmge or...Previous year question on bone cyst based on neet pg, usmle, plab and fmge or...
Previous year question on bone cyst based on neet pg, usmle, plab and fmge or...Medico Apps
 
23204922
2320492223204922
23204922radgirl
 

What's hot (20)

23205065
2320506523205065
23205065
 
23205048
2320504823205048
23205048
 
23204964
2320496423204964
23204964
 
23205032
2320503223205032
23205032
 
23204973
2320497323204973
23204973
 
23205029
2320502923205029
23205029
 
23204961
2320496123204961
23204961
 
23204958
2320495823204958
23204958
 
23205011
2320501123205011
23205011
 
23204955
2320495523204955
23204955
 
23204943
2320494323204943
23204943
 
23204916
2320491623204916
23204916
 
23204995
2320499523204995
23204995
 
23205001
2320500123205001
23205001
 
23205008
2320500823205008
23205008
 
Radiology most important signs sample questions based on neet pg , usmle, pla...
Radiology most important signs sample questions based on neet pg , usmle, pla...Radiology most important signs sample questions based on neet pg , usmle, pla...
Radiology most important signs sample questions based on neet pg , usmle, pla...
 
23204946
2320494623204946
23204946
 
Previous year question on bone cyst based on neet pg, usmle, plab and fmge or...
Previous year question on bone cyst based on neet pg, usmle, plab and fmge or...Previous year question on bone cyst based on neet pg, usmle, plab and fmge or...
Previous year question on bone cyst based on neet pg, usmle, plab and fmge or...
 
Radiology question bank
Radiology question bankRadiology question bank
Radiology question bank
 
23204922
2320492223204922
23204922
 

Viewers also liked

SLA-NY @ METRO
SLA-NY @ METROSLA-NY @ METRO
SLA-NY @ METROLisa Chow
 
Working with the Australian Curriculum: Geography, Malcolm McInerney, AGTA
Working with the Australian Curriculum: Geography, Malcolm McInerney, AGTAWorking with the Australian Curriculum: Geography, Malcolm McInerney, AGTA
Working with the Australian Curriculum: Geography, Malcolm McInerney, AGTAbecnicholas
 
Through the eyes of young observers: Geographers Imagine, Image and Create Fu...
Through the eyes of young observers: Geographers Imagine, Image and Create Fu...Through the eyes of young observers: Geographers Imagine, Image and Create Fu...
Through the eyes of young observers: Geographers Imagine, Image and Create Fu...becnicholas
 
Search Process Engineering Example
Search Process Engineering ExampleSearch Process Engineering Example
Search Process Engineering Exampleprospectcity
 
Nonverbalcommunication
NonverbalcommunicationNonverbalcommunication
NonverbalcommunicationSwati Goel
 
Central_social info
Central_social infoCentral_social info
Central_social infoandy biggin
 
HiOAs Open Access Policy
HiOAs Open Access PolicyHiOAs Open Access Policy
HiOAs Open Access PolicyTanja Strøm
 
Global-i Capabilities Presentation
Global-i Capabilities PresentationGlobal-i Capabilities Presentation
Global-i Capabilities Presentationdaveroy
 
Проекти Української мережі Євроклубів
Проекти Української мережі ЄвроклубівПроекти Української мережі Євроклубів
Проекти Української мережі ЄвроклубівSergiy Shtukarin
 
Gamification at large and in learning
Gamification at large and in learningGamification at large and in learning
Gamification at large and in learningPete Baikins
 
23204970
2320497023204970
23204970radgirl
 
Bldrux11 final
Bldrux11 finalBldrux11 final
Bldrux11 finalweitzelm
 
Land In Rouwvlag
Land In RouwvlagLand In Rouwvlag
Land In Rouwvlagwirtz
 

Viewers also liked (20)

#Curation Restart Education Project . Top 10 #edtech20 tools who will change ...
#Curation Restart Education Project . Top 10 #edtech20 tools who will change ...#Curation Restart Education Project . Top 10 #edtech20 tools who will change ...
#Curation Restart Education Project . Top 10 #edtech20 tools who will change ...
 
The Starbucks Experience Principle 4
The Starbucks Experience Principle 4The Starbucks Experience Principle 4
The Starbucks Experience Principle 4
 
SLA-NY @ METRO
SLA-NY @ METROSLA-NY @ METRO
SLA-NY @ METRO
 
WIRA brochure 2010
WIRA brochure 2010WIRA brochure 2010
WIRA brochure 2010
 
Working with the Australian Curriculum: Geography, Malcolm McInerney, AGTA
Working with the Australian Curriculum: Geography, Malcolm McInerney, AGTAWorking with the Australian Curriculum: Geography, Malcolm McInerney, AGTA
Working with the Australian Curriculum: Geography, Malcolm McInerney, AGTA
 
Through the eyes of young observers: Geographers Imagine, Image and Create Fu...
Through the eyes of young observers: Geographers Imagine, Image and Create Fu...Through the eyes of young observers: Geographers Imagine, Image and Create Fu...
Through the eyes of young observers: Geographers Imagine, Image and Create Fu...
 
Search Process Engineering Example
Search Process Engineering ExampleSearch Process Engineering Example
Search Process Engineering Example
 
Pulse Of The Market
Pulse Of The MarketPulse Of The Market
Pulse Of The Market
 
Nonverbalcommunication
NonverbalcommunicationNonverbalcommunication
Nonverbalcommunication
 
Central_social info
Central_social infoCentral_social info
Central_social info
 
HiOAs Open Access Policy
HiOAs Open Access PolicyHiOAs Open Access Policy
HiOAs Open Access Policy
 
Global-i Capabilities Presentation
Global-i Capabilities PresentationGlobal-i Capabilities Presentation
Global-i Capabilities Presentation
 
Проекти Української мережі Євроклубів
Проекти Української мережі ЄвроклубівПроекти Української мережі Євроклубів
Проекти Української мережі Євроклубів
 
Gamification at large and in learning
Gamification at large and in learningGamification at large and in learning
Gamification at large and in learning
 
Anum presentation
Anum presentationAnum presentation
Anum presentation
 
23204970
2320497023204970
23204970
 
Democratizacion
DemocratizacionDemocratizacion
Democratizacion
 
Presentation1
Presentation1Presentation1
Presentation1
 
Bldrux11 final
Bldrux11 finalBldrux11 final
Bldrux11 final
 
Land In Rouwvlag
Land In RouwvlagLand In Rouwvlag
Land In Rouwvlag
 

Similar to 23205019

23205022
2320502223205022
23205022radgirl
 
23204919
2320491923204919
23204919radgirl
 
23204937
2320493723204937
23204937radgirl
 
23205056
2320505623205056
23205056radgirl
 
Unusual Presentation of Hydatid Cyst in a Child
Unusual Presentation of Hydatid Cyst in a ChildUnusual Presentation of Hydatid Cyst in a Child
Unusual Presentation of Hydatid Cyst in a Childsemualkaira
 
Colposcopy case studies2
Colposcopy case studies2Colposcopy case studies2
Colposcopy case studies2Tariq Mohammed
 
Lesiones del cuello en la infancia
Lesiones del cuello en la infanciaLesiones del cuello en la infancia
Lesiones del cuello en la infanciaLizbet Marrero
 
23204907
2320490723204907
23204907radgirl
 
Krok 2 - 2008 Question Paper (General Medicine)
Krok 2 - 2008 Question Paper (General Medicine)Krok 2 - 2008 Question Paper (General Medicine)
Krok 2 - 2008 Question Paper (General Medicine)Eneutron
 
Presentation1.pptx, radiological imaging of female infertility.
Presentation1.pptx, radiological imaging of female infertility.Presentation1.pptx, radiological imaging of female infertility.
Presentation1.pptx, radiological imaging of female infertility.Abdellah Nazeer
 
CNS Rapid Review of Radiology
CNS Rapid Review of RadiologyCNS Rapid Review of Radiology
CNS Rapid Review of RadiologyDouble M
 
Lymphangioma in small bowel disease
Lymphangioma in small bowel diseaseLymphangioma in small bowel disease
Lymphangioma in small bowel diseaseShikha Saxena
 
Adrenal Mass in Pregnancy: Diagnostic Approach and Dilemmas
Adrenal Mass in Pregnancy: Diagnostic Approach and DilemmasAdrenal Mass in Pregnancy: Diagnostic Approach and Dilemmas
Adrenal Mass in Pregnancy: Diagnostic Approach and DilemmasApollo Hospitals
 

Similar to 23205019 (17)

23205022
2320502223205022
23205022
 
23204919
2320491923204919
23204919
 
23204937
2320493723204937
23204937
 
23205056
2320505623205056
23205056
 
Unusual Presentation of Hydatid Cyst in a Child
Unusual Presentation of Hydatid Cyst in a ChildUnusual Presentation of Hydatid Cyst in a Child
Unusual Presentation of Hydatid Cyst in a Child
 
Colposcopy case studies2
Colposcopy case studies2Colposcopy case studies2
Colposcopy case studies2
 
Lesiones del cuello en la infancia
Lesiones del cuello en la infanciaLesiones del cuello en la infancia
Lesiones del cuello en la infancia
 
ACMCR-v10-1929.pdf
ACMCR-v10-1929.pdfACMCR-v10-1929.pdf
ACMCR-v10-1929.pdf
 
Cuiello 8
Cuiello 8Cuiello 8
Cuiello 8
 
23204907
2320490723204907
23204907
 
Krok 2 - 2008 Question Paper (General Medicine)
Krok 2 - 2008 Question Paper (General Medicine)Krok 2 - 2008 Question Paper (General Medicine)
Krok 2 - 2008 Question Paper (General Medicine)
 
Neuroblastoma case scenarios for plan mangment
Neuroblastoma case scenarios for plan mangmentNeuroblastoma case scenarios for plan mangment
Neuroblastoma case scenarios for plan mangment
 
Presentation1.pptx, radiological imaging of female infertility.
Presentation1.pptx, radiological imaging of female infertility.Presentation1.pptx, radiological imaging of female infertility.
Presentation1.pptx, radiological imaging of female infertility.
 
CNS Rapid Review of Radiology
CNS Rapid Review of RadiologyCNS Rapid Review of Radiology
CNS Rapid Review of Radiology
 
Radiology 5th year, 7th lecture (Dr. Nasrin Alatrushi)
Radiology 5th year, 7th lecture (Dr. Nasrin Alatrushi)Radiology 5th year, 7th lecture (Dr. Nasrin Alatrushi)
Radiology 5th year, 7th lecture (Dr. Nasrin Alatrushi)
 
Lymphangioma in small bowel disease
Lymphangioma in small bowel diseaseLymphangioma in small bowel disease
Lymphangioma in small bowel disease
 
Adrenal Mass in Pregnancy: Diagnostic Approach and Dilemmas
Adrenal Mass in Pregnancy: Diagnostic Approach and DilemmasAdrenal Mass in Pregnancy: Diagnostic Approach and Dilemmas
Adrenal Mass in Pregnancy: Diagnostic Approach and Dilemmas
 

More from radgirl

23205042
2320504223205042
23205042radgirl
 
23205052
2320505223205052
23205052radgirl
 
23205025
2320502523205025
23205025radgirl
 
23204998
2320499823204998
23204998radgirl
 
23204989
2320498923204989
23204989radgirl
 
23204992
2320499223204992
23204992radgirl
 
23204986
2320498623204986
23204986radgirl
 
23204980
2320498023204980
23204980radgirl
 
23204983
2320498323204983
23204983radgirl
 
23204967
2320496723204967
23204967radgirl
 

More from radgirl (10)

23205042
2320504223205042
23205042
 
23205052
2320505223205052
23205052
 
23205025
2320502523205025
23205025
 
23204998
2320499823204998
23204998
 
23204989
2320498923204989
23204989
 
23204992
2320499223204992
23204992
 
23204986
2320498623204986
23204986
 
23204980
2320498023204980
23204980
 
23204983
2320498323204983
23204983
 
23204967
2320496723204967
23204967
 

Recently uploaded

VIP Hyderabad Call Girls Bahadurpally 7877925207 ₹5000 To 25K With AC Room 💚😋
VIP Hyderabad Call Girls Bahadurpally 7877925207 ₹5000 To 25K With AC Room 💚😋VIP Hyderabad Call Girls Bahadurpally 7877925207 ₹5000 To 25K With AC Room 💚😋
VIP Hyderabad Call Girls Bahadurpally 7877925207 ₹5000 To 25K With AC Room 💚😋TANUJA PANDEY
 
Call Girls Agra Just Call 8250077686 Top Class Call Girl Service Available
Call Girls Agra Just Call 8250077686 Top Class Call Girl Service AvailableCall Girls Agra Just Call 8250077686 Top Class Call Girl Service Available
Call Girls Agra Just Call 8250077686 Top Class Call Girl Service AvailableDipal Arora
 
Top Rated Hyderabad Call Girls Erragadda ⟟ 9332606886 ⟟ Call Me For Genuine ...
Top Rated  Hyderabad Call Girls Erragadda ⟟ 9332606886 ⟟ Call Me For Genuine ...Top Rated  Hyderabad Call Girls Erragadda ⟟ 9332606886 ⟟ Call Me For Genuine ...
Top Rated Hyderabad Call Girls Erragadda ⟟ 9332606886 ⟟ Call Me For Genuine ...chandars293
 
Top Rated Bangalore Call Girls Ramamurthy Nagar ⟟ 9332606886 ⟟ Call Me For G...
Top Rated Bangalore Call Girls Ramamurthy Nagar ⟟  9332606886 ⟟ Call Me For G...Top Rated Bangalore Call Girls Ramamurthy Nagar ⟟  9332606886 ⟟ Call Me For G...
Top Rated Bangalore Call Girls Ramamurthy Nagar ⟟ 9332606886 ⟟ Call Me For G...narwatsonia7
 
Call Girls Ludhiana Just Call 9907093804 Top Class Call Girl Service Available
Call Girls Ludhiana Just Call 9907093804 Top Class Call Girl Service AvailableCall Girls Ludhiana Just Call 9907093804 Top Class Call Girl Service Available
Call Girls Ludhiana Just Call 9907093804 Top Class Call Girl Service AvailableDipal Arora
 
Mumbai ] (Call Girls) in Mumbai 10k @ I'm VIP Independent Escorts Girls 98333...
Mumbai ] (Call Girls) in Mumbai 10k @ I'm VIP Independent Escorts Girls 98333...Mumbai ] (Call Girls) in Mumbai 10k @ I'm VIP Independent Escorts Girls 98333...
Mumbai ] (Call Girls) in Mumbai 10k @ I'm VIP Independent Escorts Girls 98333...Ishani Gupta
 
Call Girls Guntur Just Call 8250077686 Top Class Call Girl Service Available
Call Girls Guntur  Just Call 8250077686 Top Class Call Girl Service AvailableCall Girls Guntur  Just Call 8250077686 Top Class Call Girl Service Available
Call Girls Guntur Just Call 8250077686 Top Class Call Girl Service AvailableDipal Arora
 
♛VVIP Hyderabad Call Girls Chintalkunta🖕7001035870🖕Riya Kappor Top Call Girl ...
♛VVIP Hyderabad Call Girls Chintalkunta🖕7001035870🖕Riya Kappor Top Call Girl ...♛VVIP Hyderabad Call Girls Chintalkunta🖕7001035870🖕Riya Kappor Top Call Girl ...
♛VVIP Hyderabad Call Girls Chintalkunta🖕7001035870🖕Riya Kappor Top Call Girl ...astropune
 
Call Girls Siliguri Just Call 8250077686 Top Class Call Girl Service Available
Call Girls Siliguri Just Call 8250077686 Top Class Call Girl Service AvailableCall Girls Siliguri Just Call 8250077686 Top Class Call Girl Service Available
Call Girls Siliguri Just Call 8250077686 Top Class Call Girl Service AvailableDipal Arora
 
Call Girls Kochi Just Call 8250077686 Top Class Call Girl Service Available
Call Girls Kochi Just Call 8250077686 Top Class Call Girl Service AvailableCall Girls Kochi Just Call 8250077686 Top Class Call Girl Service Available
Call Girls Kochi Just Call 8250077686 Top Class Call Girl Service AvailableDipal Arora
 
Call Girls Aurangabad Just Call 8250077686 Top Class Call Girl Service Available
Call Girls Aurangabad Just Call 8250077686 Top Class Call Girl Service AvailableCall Girls Aurangabad Just Call 8250077686 Top Class Call Girl Service Available
Call Girls Aurangabad Just Call 8250077686 Top Class Call Girl Service AvailableDipal Arora
 
All Time Service Available Call Girls Marine Drive 📳 9820252231 For 18+ VIP C...
All Time Service Available Call Girls Marine Drive 📳 9820252231 For 18+ VIP C...All Time Service Available Call Girls Marine Drive 📳 9820252231 For 18+ VIP C...
All Time Service Available Call Girls Marine Drive 📳 9820252231 For 18+ VIP C...Arohi Goyal
 
Premium Call Girls In Jaipur {8445551418} ❤️VVIP SEEMA Call Girl in Jaipur Ra...
Premium Call Girls In Jaipur {8445551418} ❤️VVIP SEEMA Call Girl in Jaipur Ra...Premium Call Girls In Jaipur {8445551418} ❤️VVIP SEEMA Call Girl in Jaipur Ra...
Premium Call Girls In Jaipur {8445551418} ❤️VVIP SEEMA Call Girl in Jaipur Ra...parulsinha
 
The Most Attractive Hyderabad Call Girls Kothapet 𖠋 9332606886 𖠋 Will You Mis...
The Most Attractive Hyderabad Call Girls Kothapet 𖠋 9332606886 𖠋 Will You Mis...The Most Attractive Hyderabad Call Girls Kothapet 𖠋 9332606886 𖠋 Will You Mis...
The Most Attractive Hyderabad Call Girls Kothapet 𖠋 9332606886 𖠋 Will You Mis...chandars293
 
Night 7k to 12k Navi Mumbai Call Girl Photo 👉 BOOK NOW 9833363713 👈 ♀️ night ...
Night 7k to 12k Navi Mumbai Call Girl Photo 👉 BOOK NOW 9833363713 👈 ♀️ night ...Night 7k to 12k Navi Mumbai Call Girl Photo 👉 BOOK NOW 9833363713 👈 ♀️ night ...
Night 7k to 12k Navi Mumbai Call Girl Photo 👉 BOOK NOW 9833363713 👈 ♀️ night ...aartirawatdelhi
 
Call Girls Jabalpur Just Call 8250077686 Top Class Call Girl Service Available
Call Girls Jabalpur Just Call 8250077686 Top Class Call Girl Service AvailableCall Girls Jabalpur Just Call 8250077686 Top Class Call Girl Service Available
Call Girls Jabalpur Just Call 8250077686 Top Class Call Girl Service AvailableDipal Arora
 
Best Rate (Patna ) Call Girls Patna ⟟ 8617370543 ⟟ High Class Call Girl In 5 ...
Best Rate (Patna ) Call Girls Patna ⟟ 8617370543 ⟟ High Class Call Girl In 5 ...Best Rate (Patna ) Call Girls Patna ⟟ 8617370543 ⟟ High Class Call Girl In 5 ...
Best Rate (Patna ) Call Girls Patna ⟟ 8617370543 ⟟ High Class Call Girl In 5 ...Dipal Arora
 
Top Rated Bangalore Call Girls Mg Road ⟟ 9332606886 ⟟ Call Me For Genuine S...
Top Rated Bangalore Call Girls Mg Road ⟟   9332606886 ⟟ Call Me For Genuine S...Top Rated Bangalore Call Girls Mg Road ⟟   9332606886 ⟟ Call Me For Genuine S...
Top Rated Bangalore Call Girls Mg Road ⟟ 9332606886 ⟟ Call Me For Genuine S...narwatsonia7
 
Call Girls Bareilly Just Call 8250077686 Top Class Call Girl Service Available
Call Girls Bareilly Just Call 8250077686 Top Class Call Girl Service AvailableCall Girls Bareilly Just Call 8250077686 Top Class Call Girl Service Available
Call Girls Bareilly Just Call 8250077686 Top Class Call Girl Service AvailableDipal Arora
 
Premium Call Girls Cottonpet Whatsapp 7001035870 Independent Escort Service
Premium Call Girls Cottonpet Whatsapp 7001035870 Independent Escort ServicePremium Call Girls Cottonpet Whatsapp 7001035870 Independent Escort Service
Premium Call Girls Cottonpet Whatsapp 7001035870 Independent Escort Servicevidya singh
 

Recently uploaded (20)

VIP Hyderabad Call Girls Bahadurpally 7877925207 ₹5000 To 25K With AC Room 💚😋
VIP Hyderabad Call Girls Bahadurpally 7877925207 ₹5000 To 25K With AC Room 💚😋VIP Hyderabad Call Girls Bahadurpally 7877925207 ₹5000 To 25K With AC Room 💚😋
VIP Hyderabad Call Girls Bahadurpally 7877925207 ₹5000 To 25K With AC Room 💚😋
 
Call Girls Agra Just Call 8250077686 Top Class Call Girl Service Available
Call Girls Agra Just Call 8250077686 Top Class Call Girl Service AvailableCall Girls Agra Just Call 8250077686 Top Class Call Girl Service Available
Call Girls Agra Just Call 8250077686 Top Class Call Girl Service Available
 
Top Rated Hyderabad Call Girls Erragadda ⟟ 9332606886 ⟟ Call Me For Genuine ...
Top Rated  Hyderabad Call Girls Erragadda ⟟ 9332606886 ⟟ Call Me For Genuine ...Top Rated  Hyderabad Call Girls Erragadda ⟟ 9332606886 ⟟ Call Me For Genuine ...
Top Rated Hyderabad Call Girls Erragadda ⟟ 9332606886 ⟟ Call Me For Genuine ...
 
Top Rated Bangalore Call Girls Ramamurthy Nagar ⟟ 9332606886 ⟟ Call Me For G...
Top Rated Bangalore Call Girls Ramamurthy Nagar ⟟  9332606886 ⟟ Call Me For G...Top Rated Bangalore Call Girls Ramamurthy Nagar ⟟  9332606886 ⟟ Call Me For G...
Top Rated Bangalore Call Girls Ramamurthy Nagar ⟟ 9332606886 ⟟ Call Me For G...
 
Call Girls Ludhiana Just Call 9907093804 Top Class Call Girl Service Available
Call Girls Ludhiana Just Call 9907093804 Top Class Call Girl Service AvailableCall Girls Ludhiana Just Call 9907093804 Top Class Call Girl Service Available
Call Girls Ludhiana Just Call 9907093804 Top Class Call Girl Service Available
 
Mumbai ] (Call Girls) in Mumbai 10k @ I'm VIP Independent Escorts Girls 98333...
Mumbai ] (Call Girls) in Mumbai 10k @ I'm VIP Independent Escorts Girls 98333...Mumbai ] (Call Girls) in Mumbai 10k @ I'm VIP Independent Escorts Girls 98333...
Mumbai ] (Call Girls) in Mumbai 10k @ I'm VIP Independent Escorts Girls 98333...
 
Call Girls Guntur Just Call 8250077686 Top Class Call Girl Service Available
Call Girls Guntur  Just Call 8250077686 Top Class Call Girl Service AvailableCall Girls Guntur  Just Call 8250077686 Top Class Call Girl Service Available
Call Girls Guntur Just Call 8250077686 Top Class Call Girl Service Available
 
♛VVIP Hyderabad Call Girls Chintalkunta🖕7001035870🖕Riya Kappor Top Call Girl ...
♛VVIP Hyderabad Call Girls Chintalkunta🖕7001035870🖕Riya Kappor Top Call Girl ...♛VVIP Hyderabad Call Girls Chintalkunta🖕7001035870🖕Riya Kappor Top Call Girl ...
♛VVIP Hyderabad Call Girls Chintalkunta🖕7001035870🖕Riya Kappor Top Call Girl ...
 
Call Girls Siliguri Just Call 8250077686 Top Class Call Girl Service Available
Call Girls Siliguri Just Call 8250077686 Top Class Call Girl Service AvailableCall Girls Siliguri Just Call 8250077686 Top Class Call Girl Service Available
Call Girls Siliguri Just Call 8250077686 Top Class Call Girl Service Available
 
Call Girls Kochi Just Call 8250077686 Top Class Call Girl Service Available
Call Girls Kochi Just Call 8250077686 Top Class Call Girl Service AvailableCall Girls Kochi Just Call 8250077686 Top Class Call Girl Service Available
Call Girls Kochi Just Call 8250077686 Top Class Call Girl Service Available
 
Call Girls Aurangabad Just Call 8250077686 Top Class Call Girl Service Available
Call Girls Aurangabad Just Call 8250077686 Top Class Call Girl Service AvailableCall Girls Aurangabad Just Call 8250077686 Top Class Call Girl Service Available
Call Girls Aurangabad Just Call 8250077686 Top Class Call Girl Service Available
 
All Time Service Available Call Girls Marine Drive 📳 9820252231 For 18+ VIP C...
All Time Service Available Call Girls Marine Drive 📳 9820252231 For 18+ VIP C...All Time Service Available Call Girls Marine Drive 📳 9820252231 For 18+ VIP C...
All Time Service Available Call Girls Marine Drive 📳 9820252231 For 18+ VIP C...
 
Premium Call Girls In Jaipur {8445551418} ❤️VVIP SEEMA Call Girl in Jaipur Ra...
Premium Call Girls In Jaipur {8445551418} ❤️VVIP SEEMA Call Girl in Jaipur Ra...Premium Call Girls In Jaipur {8445551418} ❤️VVIP SEEMA Call Girl in Jaipur Ra...
Premium Call Girls In Jaipur {8445551418} ❤️VVIP SEEMA Call Girl in Jaipur Ra...
 
The Most Attractive Hyderabad Call Girls Kothapet 𖠋 9332606886 𖠋 Will You Mis...
The Most Attractive Hyderabad Call Girls Kothapet 𖠋 9332606886 𖠋 Will You Mis...The Most Attractive Hyderabad Call Girls Kothapet 𖠋 9332606886 𖠋 Will You Mis...
The Most Attractive Hyderabad Call Girls Kothapet 𖠋 9332606886 𖠋 Will You Mis...
 
Night 7k to 12k Navi Mumbai Call Girl Photo 👉 BOOK NOW 9833363713 👈 ♀️ night ...
Night 7k to 12k Navi Mumbai Call Girl Photo 👉 BOOK NOW 9833363713 👈 ♀️ night ...Night 7k to 12k Navi Mumbai Call Girl Photo 👉 BOOK NOW 9833363713 👈 ♀️ night ...
Night 7k to 12k Navi Mumbai Call Girl Photo 👉 BOOK NOW 9833363713 👈 ♀️ night ...
 
Call Girls Jabalpur Just Call 8250077686 Top Class Call Girl Service Available
Call Girls Jabalpur Just Call 8250077686 Top Class Call Girl Service AvailableCall Girls Jabalpur Just Call 8250077686 Top Class Call Girl Service Available
Call Girls Jabalpur Just Call 8250077686 Top Class Call Girl Service Available
 
Best Rate (Patna ) Call Girls Patna ⟟ 8617370543 ⟟ High Class Call Girl In 5 ...
Best Rate (Patna ) Call Girls Patna ⟟ 8617370543 ⟟ High Class Call Girl In 5 ...Best Rate (Patna ) Call Girls Patna ⟟ 8617370543 ⟟ High Class Call Girl In 5 ...
Best Rate (Patna ) Call Girls Patna ⟟ 8617370543 ⟟ High Class Call Girl In 5 ...
 
Top Rated Bangalore Call Girls Mg Road ⟟ 9332606886 ⟟ Call Me For Genuine S...
Top Rated Bangalore Call Girls Mg Road ⟟   9332606886 ⟟ Call Me For Genuine S...Top Rated Bangalore Call Girls Mg Road ⟟   9332606886 ⟟ Call Me For Genuine S...
Top Rated Bangalore Call Girls Mg Road ⟟ 9332606886 ⟟ Call Me For Genuine S...
 
Call Girls Bareilly Just Call 8250077686 Top Class Call Girl Service Available
Call Girls Bareilly Just Call 8250077686 Top Class Call Girl Service AvailableCall Girls Bareilly Just Call 8250077686 Top Class Call Girl Service Available
Call Girls Bareilly Just Call 8250077686 Top Class Call Girl Service Available
 
Premium Call Girls Cottonpet Whatsapp 7001035870 Independent Escort Service
Premium Call Girls Cottonpet Whatsapp 7001035870 Independent Escort ServicePremium Call Girls Cottonpet Whatsapp 7001035870 Independent Escort Service
Premium Call Girls Cottonpet Whatsapp 7001035870 Independent Escort Service
 

23205019

  • 1. 29 th Annual In-Training Examination for Diagnostic Radiology Residents Rationales February 2, 2006 Sponsored by: Commission on Education Committee on Residency Training in Diagnostic Radiology
  • 2. Section I – Pediatric Radiology Figure 1A 5. You are shown two single shot fast spin-echo images (Figures 1A and 1B) from an MR on a 22-week fetus. What is the MOST LIKELY diagnosis? A. Posterior urethral valves B. Multicystic dysplastic kidneys C. Autosomal recessive polycystic kidney disease D. Bilateral ureteropelvic junction (UPJ) obstruction 1 Diagnostic In-Training Exam 2006
  • 3. Section I – Pediatric Radiology Figure 1B 2 American College of Radiology
  • 4. Section I – Pediatric Radiology Question #5 Rationales: A. Incorrect. While patients with severe obstruction from posterior urethral valves may also show absence of amniotic fluid, posterior urethral valves lead to hydronephrosis, dilated ureters, and a markedly distended bladder. Therefore, this choice is incorrect. B. Incorrect. Although bilateral multicystic dysplastic kidneys would lead to absence of urine in the bladder, and to lack of amniotic fluid, this abnormality is rarely bilateral. Further, the kidneys show multiple, large macroscopic cysts of different size, which do not communicate. In this case, the kid- neys appear solid, since the “cysts” consist of collecting tubules, are microscopic, and the kidneys therefore appear solid. Therefore, this choice is incorrect. C. Correct. The images show symmetrical, markedly enlarged, high signal kidneys, filling the abdomen, displacing the diaphragms upwardly, small lungs, and no amniotic fluid. There is no urine in the bladder. These findings of enlarged, solid, fluid-intensity kidneys without urine production are typical of Autosomal Recessive Polycystic Kidney Disease. D. Incorrect. Although UPJ obstruction can be bilateral, and can be severe; it is rare that it would be severe enough to lead to complete lack of urine production. Furthermore, in those cases, hydronephrosis, with dilatation of the calyces, would be evident. In this case, there is no hydronephrosis. Therefore, Option D is incorrect. 3 Diagnostic In-Training Exam 2006
  • 5. Section I – Pediatric Radiology Figure 2 6. You are shown an axial T1-weighted image of a newborn term infant (Figure 2). What is the MOST LIKELY diagnosis? A. Heterotopia B. Lissencephaly C. Schizencephaly D. Hemimegalencephaly 4 American College of Radiology
  • 6. Section I – Pediatric Radiology Question #6 Rationales: A. Incorrect. Heterotopia Consists Of Gray Matter In Abnormal Locations, Thought To Be An Arrest Of Neuronal Migration Before It Reached The Brain Surface. The Most Common Locations Are Subependymal. Single Or Multiple Masses Of Gray Matter May Be Seen In The Subependymal Layer Of The Lateral Ventricles Or Subcortical White Matter. This Is Not Present In This Case, And This Choice Is Incorrect. B. Correct. The Image Shows An Abnormally Thickened, Agyric Cortex. Lissencephaly, Or “Smooth Brain” Is A Condition In Which There Is A Lack Of Gyri. MRI Features Include Abnormally Smooth Cerebral Surface, Cerebral Contour And Gray White Matter Distribution. The Cerebral Contour Is Hourglass Shaped With Shallow Sylvian Grooves. There Is Increased Gray Matter. C. Incorrect. Schizencephaly Is Believed To Result From Disordered Migration, Characterized By Abnormal Transcerebral Columns Of Gray Matter. A Cleft Extends Across The Cerebral Hemisphere From The Ventricular Surface To The Pial Surface, And Is Lined By The Abnormal Gray Matter. D. Incorrect. Hemimegalencephaly Is A Migrational Disorder Resulting In Hemihypertrophy Of The Brain. Heterotopias, Pachygyric Or Agyric Cortex May Be Present. 5 Diagnostic In-Training Exam 2006
  • 7. Section I – Pediatric Radiology Figure 3B Figure 3A 7. You are shown T1 and post-contrast, fat-saturation coronal images from an MR examination in a 3-year-old boy with urinary retention (Figures 3A and 3B). What is the MOST LIKELY diagnosis? A. Metastatic Wilms’ tumor B. Rhabdomyosarcoma C. Germ cell tumor D. Hemorrhagic cystitis 6 American College of Radiology
  • 8. Section I – Pediatric Radiology Question #7 Rationales: A. Incorrect. Wilms' Tumor Arises Peripherally In The Kidney Rather Than In Central Location And Does Not Tend To Spread Via The Ureter. Wilms' Tumor Metastases Most Commonly Occur To Local Nodes, Liver And Lung. Tumor Invasion Of The Renal Vein With Spread To The IVC And Right Atrium Also Occurs Relatively Frequently. Hemorrhage And Necrosis Within Wilms' Tumor Is Common, And Both Subcapsular And Perinephric Hematoma Can Result. However, Hemorrhage Into The Collecting System And Hematuria Is Quite Uncommon. In This Case No Renal Mass Is Evident. There Is A Large Irregular Enhancing Bladder Mass Excluding Bladder Thrombus As A Diagnosis. B. Correct. The Images Show An Enhancing Polypoid Mass In The Bladder Base And Posterior Urethra, With Secondary Bilateral Hydroureteronephrosis, And A Right Perirenal Fluid Collection Likely Representing Urinoma. Rhabdomyosarcomas In General Are More Common In Boys Than Girls With A Mean Age Of Occurrence Of 7 Years. Genitourinary Rhabdomyosarcoma Most Commonly Originates In The Prostate Or Bladder Base In Boys And The Vagina Or Bladder Base In Girls. Urinary Retention, Dysuria, Hematuria, Fecal Retention And Vaginal Mass Are The Most Common Presenting Features. This Tumor Is Also Known As Sarcoma Botryoides Because Of The Irregular Grape-Like Appearance Of The Margins Of The Tumor Extending Into The Bladder. . C. Incorrect. With The Exception Of Ovarian And Testicular Tumors, Germ Cell Tumors Tend To Occur In Very Young Children (Less Than 2 Years). Sacrococcygeal And Head And Neck Tumors Are Usually Present In The Newborn. An Occasional Site For Germ Cell Tumor Is In The Vagina In Girls, With A Mass That May Mimic Sarcoma Botryoides; However Rhabdomyosarcoma Is The Most Common Vaginal Tumor In Young Girls. Sacrococcygeal Teratomas Have Variable Internal And External Pelvic Mass Components And Typically Cause Anterior Displacement Of The Rectum And Bladder. The Images Depict A Botryoid Tumor Arising From The Bladder Base, And Therefore This Choice Is Incorrect. D. Incorrect. Hemorrhagic Cystitis Can Occur In Association With Viral Or Bacterial Infection As Well As Related To Various Drugs (Ether And Hydrogen Peroxide) And Chemotherapeutic Agents (Cyclophosphamide). There Is Usually Diffuse, Relatively Smooth Bladder Wall Thickening, Although There May Be A More Focal Mass Mimicking A Tumor, And Extensive Clot May Be Present In The Bladder. However, One Would Not Expect The Large Heterogeneous, Polypoid, Enhancing And Obstructing Mass Seen Here, Therefore This Choice Is Incorrect. 7 Diagnostic In-Training Exam 2006
  • 9. Section I – Pediatric Radiology Figure 4B Figure 4A 8. You are shown the frontal and lateral chest radiographs of a previously healthy 6- year-old child with cough and fever (Figures 4A and 4B). Which of the options below is the MOST appropriate imaging recommendation to the referring physician? A. Follow up chest radiograph B. CT scan C. Ultrasound D. No further imaging is needed 8 American College of Radiology
  • 10. Section I – Pediatric Radiology Question #8 Rationales: ALL CHOICES ARE CORRECT The finding consists of a round mass in the left paraspinal, retrocardiac region. A. The radiographs demonstrate a round opacity in the retrocardiac area, most consistent with a round pneumonia in a young child, particularly with the given history. Round pneumonias are common in children and are thought to occur because of relatively poor development of collateral air pathways for intra-alveolar spread of consolidation. When two views of the chest are obtained, frequently the lesion only appears round or mass-like in one plane and not the other. The rounded appearance of the mass is also often transient. Round pneumonias are usually associated with pneumococcal infec- tion and are most commonly located posteriorly in the perihilar or lower lobe region. In the appro- priate clinical situation the mass-like appearance of a round pneumonia should not prompt a pan- icked work up for a neoplasm. However, since other lesions can be mistaken for round pneumonia, it is considered appropriate to obtain a follow up chest x-ray to document resolution. B. The images shown demonstrate posterior left retrocardiac opacity. This may be in the middle to pos- terior mediastinum or left lower lobe. Differential considerations include round pneumonia, pul- monary sequestration, foregut duplication or neurogenic tumor. The clinical history along with the location tends to favor a round pneumonia, although no air bronchograms are seen. However air bronchograms are only present in about 30% of round pneumonias. No mass effect is seen on ribs, mediastinum or airway and the child has mild clinical symptoms. Therefore no urgent additional imaging appears necessary. Use of CT scans in children should be carefully considered both in order to obtain high quality studies that properly answer clinical questions because of the relatively high attendant radiation dose, as well as the frequent need for intravenous contrast and sedation. Clinical management and radiographic follow-up with CT obtained electively, if necessary, is a more appropriate recommendation. C. Ultrasound has been utilized in the pediatric chest especially in evaluating mediastinal or juxtamedi- astinal lesions. Studies have the advantage of being able to be performed portably in ill patients and there is no radiation exposure. However ultrasound is not usually the first choice examination to assess an abnormal chest radiographic finding, any interposing aerated lung attenuates the ultra- sound beam. The major use of chest ultrasound has been in evaluating pleural fluid collections and assessing the cystic or solid nature of accessible lesions. D. Fever is a common reason for obtaining a chest x-ray, and other lesions may be mistakenly diag- nosed as round pneumonia in a child with an acute febrile illness. Therefore it is considered appro- priate to obtain a follow up chest x-ray after resolution of the clinical illness. In this case a follow up chest x-ray demonstrated that the lesion was unchanged one month later. Further workup revealed an esophageal duplication cyst. 9 Diagnostic In-Training Exam 2006
  • 11. Section I – Pediatric Radiology Figure 5A Figure 5B 9. You are shown two images from a contrast CT image of an 8-year-old boy with leukemia who complains of pain after upper endoscopy (Figures 5A and 5B). What is the MOST LIKELY diagnosis? A. Chloroma B. Traumatic perforation of the duodenum C. Intramural duodenal hematoma D. Pancreatitis 10 American College of Radiology
  • 12. Section I – Pediatric Radiology Question #9 Rationales: A. Incorrect. Chloroma, Also Known As Granulocytic Sarcoma, Is A Localized Tumor Of Leukemic Cells. In General, Chloromas Are Uncommon, But Are A Diagnostic Consideration With Any Solid Mass In A Patient With Leukemia. Chloromas Most Commonly Involve Bone, But May Involve Other Organ Systems. A Focal Chloroma Involving Duodenum; However, Would Be Distinctly Unusual, Particularly In This Clinical Setting Of Pain After Upper Endoscopy In A Potentially Thrombocytopenic Patient. This Option Is Therefore Incorrect. B. Incorrect. The Mass Is Confined To The Wall Of The Duodenum. No Extraluminal Fluid And No Extraluminal Gas Are Seen To Suggest A Perforation Of The Duodenum. This Option Is Therefore Incorrect. C. Correct. Intramural Duodenal Hematoma May Arise From Trauma As Might Occur As A Complication Of Upper Endoscopy. Duodenal Hematomas May Also Occur From Blunt Trauma (Motor Vehicle Accidents, Bicycle Handle Bar Injuries, Child Abuse) And In Children With Bleeding Disorders. Patients With Leukemia May Be Thrombocytopenic Due To The Primary Disease Or As Consequence Of Therapy, Further Predisposing To The Development Of Duodenal Hematoma. As In This Case, The Mass Is Confined To The Duodenal Wall. The Duodenal Lumen Is Compressed By The Hematoma. This Option Is Therefore Correct. D. Incorrect. Pancreatitis May Be A Complication Of Upper Endoscopy. However, More Commonly, Pancreatitis May Be A Complication Of Endoscopic Retrograde Pancreatography (ERCP). In This Image, A Mass Is Seen Which Appears To Involve The Duodenum Rather Than The Pancreas. Only A Small Portion Of Pancreas Is Displayed On The Higher Of The Two Images, And It Appears Normal. There Is No Evidence Of Peripancreatic Edema, Or Significant Inflammatory Changes. The Diagnosis Of Pancreatitis Therefore Cannot Be Made From These Images. This Option Is Incorrect. 11 Diagnostic In-Training Exam 2006
  • 13. Section I – Pediatric Radiology Figure 6A Figure 6B 10. You are shown plain films and MR (T1 coronal and T1, fat-saturation, post-gadolinium sagittal) images of the proximal right tibia in a 12-year-old boy with a swollen and painful right leg (Figures 6A through 6C). What is the MOST LIKELY diagnosis? A. Healing stress fracture B. Brodie’s abscess C. Osteoid osteoma D. Osteosarcoma 12 American College of Radiology
  • 14. Section I – Pediatric Radiology Figure 6C 13 Diagnostic In-Training Exam 2006
  • 15. Section I – Pediatric Radiology Question #10 Rationales: A. Incorrect. A Healing Stress Fracture Demonstrates Sclerosis And Periosteal Reaction On Plain Film And Some Marrow Edema On MRI. There Is Not Extensive Mass With Enhancement, Extending Into The Surrounding Soft Tissues, As Is Seen In This Case. This Option Is Therefore Incorrect. B. Incorrect. A Brodie’s Abscess Usually Appears As A Well Defined Lytic Lesion With A Sclerotic Rim. Marrow Edema Is Seen On MRI. There Is No Soft Tissue Mass. This Option Is Therefore Incorrect. C. Incorrect. An Osteoid Osteoma Has A Central Lucent Nidus And Surrounding Sclerosis. There Is No Extension Into The Epiphysis Or Soft Tissue Mass. D. Correct. The Images Demonstrate An Aggressive, Bone-Producing Mass Involving The Proximal Right Tibia With Abnormal Periosteal Reaction. The MRI Demonstrates A Solid, Enhancing Lesion Replacing The Marrow Cavity, Crossing The Medial Physis, And An Enhancing Soft Tissue Extension Beyond The Bone. The Most Likely Diagnosis Is Osteosarcoma. 14 American College of Radiology
  • 16. Section I – Pediatric Radiology 11. A newborn infant presents on the second day of life with a distended abdomen and bilious eme- sis. A radiograph of the abdomen demonstrates markedly distended intestinal loops throughout the abdomen. What is the BEST next diagnostic examination? A. Upper GI study B. Contrast enema C. CT of the abdomen with IV contrast D. Ultrasound examination of the abdomen Question #11 Rationales: A. Incorrect. The Case Describes A Distal Bowel Obstruction In A Newborn. An Upper GI Study Would Not Be Warranted In Cases Of A Newborn With A Distal Bowel Obstruction. The Correct Test Of Choice Would Be A Contrast Enema, Preferably Performed With Water Soluble, Isotonic Contrast In Case Of A Perforation. B. Correct. The Case Describes A Distal Bowel Obstruction In A Newborn. The Correct Test Of Choice Would Be A Contrast Enema, Preferably Performed With Water Soluble, Isotonic Contrast In Case Of A Perforation. C. Incorrect. The Case Describes A Distal Bowel Obstruction In A Newborn. Abdominal CT Plays No Role In The Workup. The Correct Test Of Choice Would Be A Contrast Enema, Preferably Performed With Water Soluble, Isotonic Contrast In Case Of A Perforation. D. Incorrect. The Case Describes A Distal Bowel Obstruction In A Newborn. Abdominal Sonography Plays No Role In The Workup, And The Gas-Filled Bowel Would Render The Examination Difficult At Best. The Correct Test Of Choice Would Be A Contrast Enema, Preferably Performed With Water Soluble, Isotonic Contrast In Case Of A Perforation. 15 Diagnostic In-Training Exam 2006
  • 17. Section I – Pediatric Radiology Concerning holoprosencephaly, which one is TRUE? 12. A. The fornix and septum pellucidum are absent in all forms of holoprosencephaly. B. The posterior portion of the corpus callosum is absent in lobar holoprosencephaly. C. 10% of cases with holoprosencephaly are associated with cytogenetic abnormalities. D. Cebocephaly is most common in patients with lobar holoprosencephaly. Question #12 Rationales: A. Correct. While The Severity Of Holoprosencephaly Varies From Severe Alobar To Lobar, By Definition, All Forms Of Holoprosencephaly Do Not Have A Fornix Or Septum Pellucidum, And May Have Hypoplasia Of The Pituitary And Optic Nerves. B. Incorrect. The Cerebral Vesicles Cleave In A Dorsofrontal Direction. In Patients With Lobar And Semilobar Holoprosencephaly, The Posterior Portion Of The Corpus Callosum Is Present While The Anterior Portion Is Absent; The Entire Corpus Callosum May Be Present In Lobar Holoprosencephaly. C. Incorrect. Up To 40% Of Cases Are Linked To Chromosomal Anomalies, 20% With Syndromes, With Only 40% Cases Isolated. D. Incorrect. Severe Facial Anomalies, Such As Cyclopia, Cebocephaly And Ethmocephaly, Are Most Likely In Patients With Alobar Holoprosencephaly. However, A Normal Face Does Not Exclude A Severe Form Of Holoprosencephaly. 16 American College of Radiology
  • 18. Section I – Pediatric Radiology Which one of the following findings is seen in most patients with Dandy 13. Walker malformation? A. Intact vermis B. Small cisterna magna C. Elevated tentorium cerebelli D. Agenesis of the corpus callosum Question #13 Rationales: A. Incorrect. By Definition, The Most Characteristic Finding Of Dandy Walker Malformation Is Complete Or Partial Absence Of The Cerebellar Vermis. B. Incorrect. Fourth Ventricle Communicates With An Enlarged Cisterna Magna Bordered By The Cerebellar Hemispheres. C. Correct. The Tentorium And Torcula Are Elevated With Dandy Walker Malformation, Best Evaluated On Midline Sagittal Images Of The Brain. On Plain Films, The Anomaly Is Characterized By Lambdoid-Torcular Inversion. D. Incorrect. Dysgenesis Of The Corpus Callosum Occurs In Only 20-25% Of Patients. Gyral Abnormalities Also Occur In A Small Number Of Patients With Dandy Walker Malformation. 17 Diagnostic In-Training Exam 2006
  • 19. Section I – Pediatric Radiology Concerning periventricular leukomalacia, which one is CORRECT? 14. A. Injury is secondary to venous infarction. B. Areas of cystic necrosis develop 1-3 weeks after injury. C. Infants are at greatest risk for PVL at 36-38 weeks gestation. D. Ultrasound is sensitive in the identification of the acute lesions. Question #14 Rationales: A. Incorrect. Injury is thought to be arterial watershed infarction of the premature infant’s periventricu- lar white matter. Venous infarction is believed to be the mechanism of the parenchymal hemorrhagic component in Grade IV germinal matrix hemorrhage. B. Correct. The multicystic periventricular components, characteristic at sonography, represent areas of cystic necrosis in ischemic areas, and develop 1-3 weeks after the acute injury, after resolution of the initially echodense lesions. C. Incorrect. Injury typically occurs in preterm infants less than 32 weeks gestation. D. Incorrect. Ultrasound is often insensitive in the acute stage, with as many as 50% of initial studies in the first few days following injury reported as normal in appearance. 18 American College of Radiology
  • 20. Section I – Pediatric Radiology 15. Which one of the following is characterized by the finding of a microcolon? A. Duodenal atresia B. Meconium plug syndrome C. Meconium ileus D. Short-segment Hirschsprung’s disease Question #15 Rationales: A. Incorrect. In patients with duodenal atresia, sufficient succus entericus is produced to allow normal colonic caliber. B. Incorrect. Mecomiun plug syndrome is associated with dysmotility of the colon; infants are typical- ly full term, often with maternal history of preeclampsia. Colon is normal in caliber and filled with meconium plugs. Once the meconium is extruded, the patient is relieved of symptoms. C. Correct. Meconium ileus, associated with cystic fibrosis, is the result of inspissation of tenacious meconium in the ileum, which prevents passage of meconium into the colon. The colon is not used prenatally, resulting in a microcolon. D. Incorrect. Hirschsprung’s disease is characterized by a distal aperistaltic segment, typically of nor- mal caliber, with a more dilated proximal bowel as the transition zone develops. The abnormal seg- ment may demonstrate a serrated appearance, believed to result from uncoordinated, aperistaltic contractions. Suction biopsy can confirm the diagnosis. 19 Diagnostic In-Training Exam 2006
  • 21. Section I – Pediatric Radiology Concerning the presence of a perirenal fluid collection in a newborn infant, which one is 16. CORRECT? A. This finding is associated with posterior urethral valves. B. The finding represents sympathetic transudation of fluid due to obstruction. C. Secondary infection is not a significant concern. D. Early post-contrast CT or MR images are diagnostic. Question #16 Rationales: A. Correct. Long standing high grade obstruction in utero is associated with renal dysplasia and poor renal function. The presence of loculated or free urine extravasation from the kidney of a newborn infant is most commonly associated with posterior urethral valves, and is secondary to forniceal rupture and decompression into the perirenal space, at times extending into the peritoneal cavity, presenting as urinary ascites. This finding has been associated with relative preservation of function in the kidney, probably due to decompression of the obstruction. B. Incorrect. This fluid collection results from forniceal rupture of a calyx and extravasation of urine forming a subcapsular urinoma. Subcapsular, perinephric and paranephric urinomas can occur in association with urinary obstruction, including posterior urethral valves, ureteropelvic junction obstruction, neurogenic bladder and bladder tumors or after trauma to the urinary tract. The collec- tion may rupture into the peritoneal space producing urinary ascites. C. Incorrect. Secondary infection of fluid collections in a newborn is always of concern, especially when there is underlying obstruction and/or reflux. D. Incorrect. If a contrasted examination (CT or MRI) were done, delayed images would be most use- ful in demonstrating contrast accumulation within the urinoma, confirming the connection to the urinary tract. In the setting of posterior urethral valves, however, upper tract contrast examinations are unnecessary for diagnosis. 20 American College of Radiology
  • 22. Section I – Pediatric Radiology 17. Concerning leakage radiation for a unit used for pediatric chest filming, what is the maximum leakage radiation allowed at 1 meter from the x-ray tube source when the system is operated at the maximum continuous allowable tube current and kilovoltage? A. 2 mR/week B. 10 mR/hour C. 100 mR/hour D. No specified limits Question #17 Rationales: A. Incorrect. This is maximum permissible dose limit outside an x-ray room. B. Incorrect. See correct answer. C. Correct. According to NCRP report no. 91, the allowed maximum limit for leakage radiation is 100 mR/hr at 1 meter when the system is operated at maximum continuous mA and kVp. D. Incorrect. See correct answer. 21 Diagnostic In-Training Exam 2006
  • 23. Section I – Pediatric Radiology 18. Which of the following radiographic findings is MOST useful in excluding the diagnosis of hyaline membrane disease in a newborn infant? A. Pleural effusion B. Hyperinflation C. Unossified humeral epiphyses D. Asymmetric pulmonary opacity Question #18 Rationales: A. Incorrect. While pleural effusion is an uncommon finding in hyaline membrane disease, its presence or absence is not specific in excluding the diagnosis. Small pleural effusion may be seen in transient tachypnea of the newborn which can co-exist with hyaline membrane disease, especially on early radiographs. Although in the appropriate clinical setting, the presence of diffuse ground glass opaci- ty with accompanying pleural effusion should raise the question of infection, particularly group B streptococcal pneumonia, pleural effusion alone cannot exclude hyaline membrane disease. B. Correct. The underlying cause of hyaline membrane disease is deficiency of surfactant; this lipoprotein complex produced by Type II pneumocytes coats alveoli and prevents them from col- lapsing in expiration. Diffuse alveolar atelectasis with pulmonary underaeration is the hallmark of this disease. Increased lung volumes would strongly suggest excluding hyaline membrane disease as a diagnostic possibility. With early intubation and administration of exogenous surfactant, character- istic radiographic findings may be less marked and more transient. C. Incorrect. Ossification of humeral epiphyses may be used as a sign of skeletal maturity on a chest radiograph. Ossified humeral epiphyses are not seen in infants less than thirty-six weeks of gesta- tion and are present in approximately 40% of term infants. Therefore the presence of humeral epi- physes would diminish the likelihood of hyaline membrane disease, a disease of prematurity. Unossified epiphyses would not be useful in excluding the disease. D. Incorrect. While diffuse symmetric ground glass opacity is the characteristic feature of hyaline membrane disease, some variation of this pattern is not uncommon. Lower lobe involvement tends to be more marked than upper lobe; opacity may be more marked on the right than on the left side. After surfactant administration there is often asymmetric clearing of the lungs. Therefore, the pres- ence of asymmetry does not exclude the diagnosis of hyaline membrane disease. 22 American College of Radiology
  • 24. Section I – Pediatric Radiology Concerning the thymus in childhood, which one is CORRECT? 19. A. Hodgkin’s disease rarely involves the thymus. B. Normal thymus obscures the great vessels at sonography. C. Thymic cysts may occur in the neck. D. Normal thymus is bright on T1- and dark on T2-weighted MR images. Question #19 Rationales: A. Incorrect. Hodgkin’s disease often occurs in the anterior mediastinum and typically infiltrates the thymus. Bulky adjacent mediastinal adenopathy usually occurs. On CT imaging, the thymus is enlarged, nodular and heterogeneous with areas of necrosis frequently present, as opposed to the smooth homogenous normal thymus. Compression of the airway is often a significant concern. B. Incorrect. The normal thymus is of uniform, midi-level echogenicity, and offers an excellent win- dow into the mediastinum, by displacing surrounding air-filled lung. C. Correct. Thymic cysts are most commonly found in thymic rests in the neck, rather than in the main gland. These may present acutely due to infection or hemorrhage. They may be associated with bone marrow aplasia and have been reported in patients with HIV and treated lymphoma. D. Incorrect. On MR images the normal thymus in childhood is relatively dark (slightly hyperintense to muscle) on T1 weighted images and has uniform bright signal on T2 weighted sequences. In younger infants the thymus has a quadrilateral shape with convex borders, becoming more triangu- lar in children over five years of age. Fatty infiltration of the thymus is usually not seen until the late teenage years or adulthood. 23 Diagnostic In-Training Exam 2006
  • 25. Section I – Pediatric Radiology Concerning imaging of a child with a bronchial foreign body, which one is CORRECT? 20. A. A decubitus radiograph with the affected side up will be helpful in demonstrating ipsilateral air trapping. B. A nuclear medicine ventilation perfusion scan will likely demonstrate symmetric lung perfusion but asymmetric ventilation. C. On fluoroscopy the mediastinum will shift away from the affected side in expiration. D. The majority of foreign bodies are radiopaque and visible on chest radiographs. Question #20 Rationales: A. Incorrect. When placed in the decubitus position, the dependent (down) lung deaerates and the non- dependent lung expands. Therefore to demonstrate air trapping i.e. lack of dependent deflation, the useful decubitus view is with the affected side down. B. Incorrect. When the lung is abnormally ventilated there is also decreased perfusion due to reflex vasoconstriction. There may be very little perfusion of the affected side when significant air trap- ping is present. C. Correct. This is one of the cardinal observations in fluoroscopic evaluation of a foreign body. On inspiration the mediastinum is in its normal location i.e. central. With expiration the mediastinum shifts towards the normal lung which deflates, and away from the persistently hyperinflated abnor- mal lung. Other fluoroscopic observations include lack of deflation and decreased diaphragmatic excursion on the affected side D. Incorrect. Only a small number of foreign bodies are radiopaque. The most common endobronchial foreign bodies are ingested foods, such as peanuts and raw carrots. Therefore the radiographic fea- tures rely on secondary findings, predominantly air trapping distal to the foreign body due to partial obstruction of the affected airway. With a larger or more long standing foreign body, pulmonary opacity due to atelectasis or post obstructive pneumonia may occur. 24 American College of Radiology
  • 26. Section I – Pediatric Radiology Concerning Tetralogy of Fallot, which one is CORRECT? 21. A. A right-sided aortic arch is present in approximately 60% of cases. B. The four features consist of ASD, VSD, overriding aorta and right ventricular outflow tract obstruction. C. Radiographs typically demonstrate marked cardiomegaly with a “boot shaped” heart. D. Right-sided obstruction may include infundibular, valvular and peripheral pulmonic stenoses. Question #21 Rationales: A. Incorrect. Tetralogy of Fallot is one of the most frequent lesions to be associated with a right-sided aortic arch. This occurs however in approximately 20 - 30% of cases. Other cardiac lesions associat- ed with a right-sided aortic arch include truncus arteriosus (30%), tricuspid atresia (10 - 15%) and double outlet right ventricle (12%). B. Incorrect. ASD is not one of the four cardinal features in Tetralogy of Fallot. Right ventricular hypertrophy is the fourth feature in addition to VSD, overriding aorta and right ventricular outflow obstruction. ASD occurs in approximately 15% of cases; when present the entity is termed Pentalogy of Fallot. C. Incorrect. Although the quot;boot shapedquot; heart is the classically described configuration in Tetralogy of Fallot, preoperatively the heart is usually normal in size, or only mildly enlarged. The “boot-shaped heart” is produced by the combination of a concave main pulmonary artery segment and an elevated cardiac apex, indicative of right ventricular hypertrophy without dilatation and cardiac enlargement. D. Correct. Right-sided obstruction is often at multiple levels. Typically there is infundibular obstruc- tion without or with valvular stenosis, or even atresia. Peripheral pulmonary artery coarctation, uni- lateral hypoplasia or absence of a pulmonary artery also occurs, most often on the left side. Conversely there may be complete absence of the pulmonary valve leaflets with marked pulmonic regurgitation and very large central pulmonary arteries. 25 Diagnostic In-Training Exam 2006
  • 27. Section I – Pediatric Radiology Concerning slipped capital femoral epiphysis, which one is CORRECT? 22. A. The frog lateral radiograph is contraindicated B. Produces knee pain in one quarter of patients C. Is bilateral in 75 – 80% of cases D. Is most common in athletes Question #22 Rationales: A. Incorrect. The femoral head slips posteriorly and medially. The frog lateral view is essential to detect the posterior component of the slip, which may be the major component in some patients. B. Correct. Patients with slipped capital femoral epiphysis usually present with hip or groin pain; however, in approximately 25% of patients pain is referred to the ipsilateral knee. These patients may present with knee pain alone or knee pain in combination with hip/groin pain. In a patient in the typical age range for slipped capital femoral epiphysis, unexplained knee pain should prompt consideration for slipped capital femoral epiphysis. C. Incorrect. Slipped capital femoral epiphysis is bilateral in approximately one quarter to one third of patients, much less than 75-80%. D. Incorrect. Slipped capital femoral epiphysis is most common in obese adolescents. 26 American College of Radiology
  • 28. Section I – Pediatric Radiology Concerning Ewing sarcoma of bone, which one is CORRECT? 23. A. It is most common in children under 5 years of age. B. It is the most common primary bone malignancy in childhood. C. The more aggressive tumors produce more osteoid. D. Histologically it is identical to primitive neuroectodermal tumor (PNET). Question #23 Rationales: A. Incorrect. Although Ewing sarcoma can occur in a child under 5 years of age, it is relatively rare in this age group. Ewing sarcoma is most common in the second decade of life. B. Incorrect. Ewing sarcoma is the second most common primary bone malignancy in childhood. Osteosarcoma is considerably more common. C. Incorrect. Ewing sarcoma does not produce osteoid. Osteoid is produced by osteosarcoma. A scle- rotic response may be seen within the host bone and up to 15% of Ewing sarcomas will thus appear sclerotic at presentation; however, the tumor itself does not produce osteoid. D. Correct. Ewing sarcoma and PNET are histologically identical. Moreover, they are cytogenetically identical and now considered by pathologists to be the same tumor. 27 Diagnostic In-Training Exam 2006
  • 29. Section I – Pediatric Radiology 24. Concerning the conversion of hematopoietic bone marrow to fatty bone marrow in the femur of a child, which portion of the femur is the LAST to convert? A. Proximal femoral epiphysis B. Proximal femoral metaphysis C. Diaphysis D. Distal femoral metaphysis Question #24 Rationales: A. Incorrect. The epiphyses almost immediately convert from hematopoietic marrow to fatty marrow as soon as they are ossified. B. Correct. The epiphyses convert to fatty marrow soon after they ossify. Diaphysis precedes metaph- ysis and distal precedes proximal. Therefore, within the femurs, the proximal metaphyses are the last portion of the bone to convert from hematopoietic marrow to fatty marrow. Commonly in teenagers, residual patchy hematopoietic marrow is still present within the proximal femoral meta- physis on MR imaging. C. Incorrect. The diaphysis converts from hematopoietic marrow to fatty marrow before the metaphy- ses. D. Incorrect. Conversion proceeds from distal to proximal. The distal femoral metaphyses therefore convert from hematopoietic marrow to fatty marrow prior the proximal femoral metaphyses. 28 American College of Radiology
  • 30. Section I – Pediatric Radiology 25. To keep patient dose as low as reasonably achievable for digital radiography examinations using a CR or DR detector, the mAs is selected to provide acceptable: A. image contrast. B. optical density. C. luminance on the image monitor. D. signal to noise ratio. Question #25 Rationales: A. Incorrect. Digital detectors are inherently limited by signal to noise ratio, and contrast is variable according to digital grayscale translation tables (windowing and leveling the image). B. Incorrect. There is no optical density measure with digital images. For analog screen-film images this, of course, is what is required. C. Incorrect. The luminance of the monitor is independent of the characteristics of the image. Also, for digital images, the overall signal intensity of the output image is scaled so that the proper range of signals is displayed (or at least they can be adjusted to the appropriate luminance range) D. Correct. Digital detectors are inherently signal to noise ratio (SNR) limited, because of the ability to vary the image display characteristics by post-processing (contrast enhancement, spatial resolu- tion enhancement, etc.) The mAs is linearly related to the number of x-ray photons on the patient and also to the number of x-ray photons absorbed by the x-ray detector. Therefore, the SNR (N/√N = √N) is determined by the number of x-rays detected and converted to an output signal. Even though digital systems have the ability to enhance contrast, noise will also be enhanced and the SNR will remain constant. So, the ability to detect a subtle finding in a noisy background for a digital image depends on the required SNR for a given examination. The mAs should be chosen at a level that gives just enough (not too much) radiation such that the SNR will be high enough to make the diagnosis. 29 Diagnostic In-Training Exam 2006
  • 31. Section I – Pediatric Radiology Which one of the following is a characteristic feature of renal duplication in girls? 26. A. Reflux into the upper system B. Ectopic lower pole ureter C. Incontinence D. Duplication of the uterus Question #26 Rationales: A. Incorrect. Although reflux may occur into either the upper or lower moieties of an uncomplicated duplication, the typical consequence of duplication is obstruction of the ectopic upper pole ureter, and reflux into the lower moiety, likely secondary to interference of the abnormal upper pole ureter with the normal antireflux tunnel mechanism of the orthotopic lower pole. B. Incorrect. The lower pole ureter inserts normally at the trigone. The upper pole ureter arises above the normal ureteric bud, and therefore inserts ectopically, medially and inferiorly within the bladder, or along the course of the mesonephric duct. Therefore, option B is false. C. Correct. The ureters are derivatives of the mesonephric duct, and therefore ectopic ureters insert along its derivatives or remnants. In boys, the ectopic ureter will always insert above the sphincter, medially into the bladder, or into the seminal vesicles. However, in girls with duplex kidney, the ectopic ureter may insert below the sphincter, into the urethra, or into the vagina, resulting in uri- nary incontinence. This is characterized by constant dampness. Therefore, option C is correct. D. Incorrect. Duplication of the uterus, with obstruction of one of the two systems, is typically associ- ated with ipsilateral renal agenesis or dysgenesis. However, renal duplications are not typically associated with duplication of the uterus. 30 American College of Radiology
  • 32. Section I – Pediatric Radiology Regarding duodenal obstruction, which one is CORRECT? 27. A. Plain films may be normal in patients with volvulus. B. Nonbilious vomiting excludes the diagnosis. C. Atresia is secondary to intrauterine ischemia. D. Windsock duodenum is typically found in neonates. Question #27 Rationales: A. Correct. Volvulus obstructs at the third portion of the duodenum, and presents with bilious vomit- ing. Volvulus occurs about the axis of the superior mesenteric artery, and leads to ischemia of its territory, from the duodenojejunal junction to the splenic flexure. Therefore, insufficient time elaps- es for marked duodenal dilatation to occur, and the plain films may be normal, or demonstrate rela- tive paucity of distal gas with longer or more severe obstruction. Absence of bowel distension may lead to a false sense of security and diminished concern for obstruction. B. Incorrect. Although most cases of duodenal atresia or stenosis occur distal to the ampulla of Vater, the obstruction may lie proximal to the ampulla in a minority of patients, in whom the emesis may be nonbilious. Therefore, nonbilious vomiting does not exclude the diagnosis, and this option is incorrect. C. Incorrect. Although small bowel and colonic atresias are considered to be secondary to an intrauter- ine ischemic event, duodenal atresia is believed to be the result of failure of recanalization. Therefore this option is incorrect. D. Incorrect. Windsock duodenum is the result of stretching of duodenal membrane in patients with membranous duodenal stenosis. Therefore, it is typically seen in older patients. 31 Diagnostic In-Training Exam 2006
  • 33. Section I – Pediatric Radiology Regarding pyloric stenosis, which one is TRUE? 28. A. It is present at birth. B. Affected infants present with paradoxical aciduria. C. Paternal history is a greater risk factor than maternal history. D. Affected infants have no appetite. Question #28 Rationales: A. Incorrect. Pyloric stenosis develops after birth, and typically presents in infants at 4-12 weeks of age. B. Correct. Vomiting leads to dehydration and waste of sodium, potassium and hydrochloric acid. Renal mechanisms supervene, with conservation of sodium at the expense of hydrogen ion, leading to a paradoxical aciduria in a patient with hypochloremic alkalosis. C. Incorrect. Male and female children of affected mothers carry a 20% and 7% respective risk of developing the condition, whereas male and female children of affected fathers carry a respective risk of 5% and 2.5%. D. Incorrect. Despite the distended stomach, the infants are malnourished, and exhibit a voracious appetite, typically gnawing at their hands in an effort to obtain nourishment. 32 American College of Radiology
  • 34. Section I – Pediatric Radiology 29. Which of the following conditions is associated with pulmonary hypoplasia? A. Bochdalek hernia B. Potter syndrome C. Vascular ring D. Tracheal bronchus Question #29 Rationales: A & B ARE BOTH CORRECT A. Correct. The posterolateral Bochdalek diaphragmatic hernia is usually large and associated with considerable mass effect on the developing lung. The anteromedial Morgagni hernia is typically small and most often asymptomatic. Bochdalek hernia is strongly associated with pulmonary hypoplasia whereas hypoplasia is usually not a feature of the Morgagni defect. Occasionally Morgagni hernia may be large or bilateral often with herniation into the pericardial sac with car- diorespiratory compromise. B. Correct. Potter syndrome was originally described as renal agenesis in association with fetal anuria and oligohydramnios. Bilateral pulmonary hypoplasia is thought to be caused by extrinsic compres- sion of the fetal thorax by the uterus in association with decreased amniotic fluid. Other causes of olgohydramnios including bilateral urinary obstruction, bilateral cystic or dysplastic kidneys and prolonged amniotic fluid leak have also been found to be associated with pulmonary hypoplasia. Thoracic cage compression, most commonly due to bone dysplasia and intrathoracic lung compres- sion, notably associated with congenital diaphragmatic hernia, pleural fluid or a large thoracic or abdominal mass are also causes of bilateral pulmonary hypoplasia. Additional etiologic factors in bilateral pulmonary hypoplasia are conditions associated with decreased fetal breathing (e.g. myotonic dystrophy, myasthenia gravis, hypotonia) and decreased pulmonary blood flow, such as in Tetralogy of Fallot and pulmonary atresia. C. Incorrect. Vascular ring is a result of abnormal vascular and ligamentous mediastinal structures, with encirclement of the trachea and esophagus. The symptomatic vascular ring typically causes compression of the trachea and presents with respiratory distress and repeated episodes of pneumo- nia. However, this abnormality is not associated with pulmonary hypoplasia. D. Incorrect. A tracheal bronchus describes the situation where the right upper lobe, or one of its seg- ments, is supplied directly from the trachea, rather than as a branch of the right mainstem bronchus. When the bronchus supplies the entire lobe, it is termed a “bronchus sui” or pig bronchus, as this is the normal porcine anatomical arrangement. The tracheal bronchus may be asymptomatic and dis- covered incidentally, or may be associated with repeated right upper lobe infections. Its incidence is reported as 0.1 – 2% of the population, and is associated with rib anomalies and Down syndrome. However, it is not associated with pulmonary hypoplasia. 33 Diagnostic In-Training Exam 2006
  • 35. Section I – Pediatric Radiology 30. Regarding pulmonary sling, which of the following is TRUE? A. The right pulmonary artery arises aberrantly from the left. B. The abnormal artery crosses posterior to the esophagus. C. Pulmonary expansion on the chest radiograph is usually normal. D. It can be complicated by tracheal stenosis. Question #30 Rationales: A. Incorrect. The pulmonary artery sling consists of abnormal origin of the left pulmonary artery from the right. The aberrant left artery does not arise from the main pulmonary artery at the bifurcation, but from the right branch; it then courses posteriorly, and turns to the left between the trachea and the esophagus to the left hilum. Therefore, this choice is incorrect. B. Incorrect. As stated above, the course of the aberrant pulmonary artery lies between the trachea and the esophagus, therefore it courses anterior to the esophagus. This is in contradistinction to vascular rings, where the abnormal vessel courses posterior to the esophagus, encircling both the trachea and esophagus within the vascular ring. C. Incorrect. Plain chest radiographs on patients with pulmonary artery sling typically show abnormal inflation, with hyperinflation or asymmetrical inflation of the lungs. There may be a low position of the left hilum, as the normal epibronchial left pulmonary artery is absent. Anterior bowing of the trachea is often seen on the lateral view, as the aberrant vessel courses between the trachea and the esophagus. D. Correct. Tracheal stenosis may occur at the site of the crossing vessel, or may be part of a more generalized anomaly with complete cartilage rings. The anomaly is typically symptomatic early in life, with infants presenting with respiratory stridor, feeding difficulties, and repeated respiratory infections. 34 American College of Radiology
  • 36. Section I – Pediatric Radiology 31. Which of the following is a Salter IV fracture? A. Lateral condylar fracture of the distal humerus B. Avulsion of the medial epicondyle of the distal humerus C. Juvenile Tillaux fracture D. Bucket handle fracture of non-accidental trauma Question #31 Rationales: A & C ARE BOTH CORRECT A. Correct. The lateral condylar fracture extends through distal humeral metaphysis, distal humeral growth plate and distal humeral epiphysis. This is a Salter IV fracture. When the epiphyseal portion of the fracture passes through cartilage and is not seen on radiographs, it can be delineated by MR or ultrasound. If the fracture is complete there may be displacement of the lateral condylar frag- ment. If the fracture through the epiphyseal cartilage is incomplete, there may be minimal or no dis- placement of the lateral condylar fragment. B. Incorrect. An avulsion fracture of the medial epicondyle goes through the growth plate of the medi- al epicondylar apophysis. This is a Salter I fracture. C. Correct. Juvenile Tillaux fracture is a Salter III fracture. The fracture line course through epiphysis, usually in a sagittal or oblique sagittal plane, and through the lateral portion of growth plate. This fracture occurs near puberty when the distal tibial growth plate is beginning to fuse. Fusion begins medially. The Tillaux fracture involves the lateral aspect of the growth plate which is not yet fused. D. Incorrect. The “bucket handle” fracture of non-accidental trauma does not involve the growth plate and is not classified by the Salter-Harris system. Histologically, Kleinman et al have shown that this fracture is a transverse fracture through the metaphysis close to the growth plate. 35 Diagnostic In-Training Exam 2006
  • 37. Section I – Pediatric Radiology 36 American College of Radiology